HESI Exam Prep

अब Quizwiz के साथ अपने होमवर्क और परीक्षाओं को एस करें!

Which instruction would the nurse include when providing medication teaching to a client prescribed trimethoprim-sulfamethoxazole for cystitis? A. "Drink eight to ten glasses of water daily." B. "Take this medication with orange juice." C. "Take the medication with meals." D. "Take the medication until symptoms subside."

A Rationale: A urinary output of a least 1500 mL daily should be maintained to prevent crystalluria (crystals in the urine). Taking the medication with orange juice provides no advantage. Also, orange juice produces an alkaline ash, which resuts in an alkaline urine that supports the growth of bacteria. Trimethoprim-sulfamethoxazole should be taken 1 hour before meals for maximum absorption. A prescribed course of antibiotics must be completed to eliminate the infection, which can exist on a subclinical level after symptoms subside

Which action describes a therapeutic effect of atenolol? A. Heart rate decreases B. Blood pressure increases C. Bronchospasm is relieved D. Pulse oximetry improves

A Rationale: Atenolol, a beta-blocker, slows the rate of sinoatrial (SA) node discharge and atrioventricular (AV) node conduction, thus decreasing the heart rate; it prevents angina by decreasing the cardiac workload and myocardial oxygen consumption. Blood pressure is not increased and may be decreased. Atenolol may promote bronchospasm, not relieve it. Atenolol does not directly affect gas exchange in the lungs to promote improving oxygenation

One day ago, a client underwent extraction of a tooth because of an underlying tumor. Which health care professional is appropriately involved in caring for the postoperative oral hygiene needs of this client? A. Registered nurse (RN) B. Licensed practical nurse (LPN) C. Licensed vocational nurse (LVN) D. Unlicensed assistive personnel (UAP)

A Rationale: If there is any anticipated risk in client care, the task should not be delegated by should be carried out by the RN. Because there is a risk of infection and oral bleeding, the RN would not delegate the task of postoperative oral hygiene to the LPN, LVN, or UAP

Which observation supports the nurse's suspicion that a client may have myoglobinuria? A. Red-colored urine B. Brown-colored urine C. Dark, amber-colored urine D. Very pale, yellow-colored urine

A Rationale: Red-colored urine in clients with kidney dysfunction indicates the presence of myoglobin. Brown-colored urine indicates increased bilirubin levels. Dark, amber-colored urine indicates concentrated urine. Very pale, yellow urine indicates dilute urine

Which goal would the nurse identify for a toddler with dehydration caused by diarrhea? A. Improvement of fluid balance B. Continuation of an antidiarrheal diet C. Administration of antimicrobial treatments D. Retention of weight appropriate for height

A Rationale: Rehydration and correction of electrolyte imbalances are the priorities; diarrhea causes loss of fluids and electrolytes that can be life threatening. Antidiarrheal diets are no longer prescribed for children with diarrhea. Oral rehydration therapy is the treatment of choice. There is no indication for antimicrobials; most sources of diarrhea are viral. There are no data to indicate that the child is overweight or underweight

A pediatric client is prescribed an intravenous infusion of methylprednisolone. Which clinical manifestation requires immediate intervention during administration of the initial dose? A. Polyuria B. Tinnitus C. Drowsiness D. Hypotension

A Rationale: Intravenous administration of a steroid can cause a rapid increase in the blood glucose level. One early sign of hyperglycemia is increased urine output. Blood glucose should be checked frequently, and insulin administered as needed. Tinnitus is associated with some antibiotics and with aspirin, not steroids. Drowsiness is associated with sedatives, not steroids. Hypertension, not hypotension, is associated with steroid administration

Which example illustrates health promotion activities? Select all that apply. A. Good nutrition B. Regular exercise C. Physical awareness D. Immunization against measles E. Education about stress management

A, B Rationale: Health promotion activities enable clients to enhance or maintain their current health levels. Good nutrition and regular exercise are examples of such activities. Immunization against measles is an example of an illness prevention activity. Education about stress management and physical awareness are examples of a wellness education activity

Which assessment finding during the administration of intravenous penicillin would prompt the nurse to stop the infusion? Select all that apply. A. Hives B. Itching C. Nausea D. Skin rash E. Shortness of breath

A, B, D, E Rationale: Penicillin administration carries a high rate of allergic reaction, so the nurse monitors the client for signs of allergy. Hives, itching, skin rash, and shortness of breath are all indications of allergic reaction and warrant cessation of the infusion and contact with the health care provider. Nausea is not an indication of allergic reaction

A client suffers hypoxia and a resultant increase in deoxygenated hemoglobin in the blood. Which is/are the best site(s) to assess this condition? Select all that apply. A. Lips B. Sclera C. Mouth D. Sacrum E. Nail beds F. Shoulders

A, C, E Rationale: Prolonged hypoxia resulting in increased amounts of deoxygenated blood causes cyanosis, which can be best evaluated in lips, mouth, nail beds, and skin (in extreme conditions). Sclera is the site of assessment for jaundice, whereas shoulders are assessed to confirm the condition of erythema

While assessing a client's hair, the nurse notices the client has head lice. The nurse teaches the client about hair hygiene and lice control. Which client statement indicates an understanding of the teaching? Select all that apply. A. "I will clean my comb in ammonia water." B. "I should use lindane-containing shampoo." C. "I should shampoo my hair in a tub or shower." D. "I should use a dilute vinegar solution to loosen the nits." E. "I should use a shampoo treatment once every 24 hours."

A, D, E Rationale: Lindane may be used to treat lice and scabies, but it may cause serious side effects. Clients with live are instructed not to wash hair in a tub or shower because this action may cause the lice to migrate to other sites. Ammonia water should be used to clean combs and other hair accessories to enhance lice control. Nits are loosened by the use of dilute vinegar solution. Shampooing should be continued once every 24 to 48 hours.

The nurse is providing education about excellent food sources of vitamin A for a client who is deficient in this vitamin. Which foods would the nurse include in the teaching? Select all that apply. A. Carrots B. Oranges C. Tomatoes D. Leafy greens E. Cantaloupe

A, D, E Rationale: Yellow and orange vegetables, such as carrots, contain large quantities of vitamin A in human nutrition. Cantaloupe, sweet potatoes, and apricots also are high in vitamin A. Dark-green leafy vegetables contain large quantities of vitamin A in human nutrition. Oranges are considered a good source of both vitamin C and potassium. Tomatoes are a good source of vitamin C

Which factor would the nurse assess for a client reporting constipation? Select all that apply. A. Diet B. Fluid intake C. Use of laxatives D. Date of last bowel movement E. Use of opioid pain medications

All of the above Rationale: If a client complains of constipation, the nurse would inquire about factors related to constipation including diet, fluid intake, laxative use, date of last bowel movement, and whether or not the client is taking opioid pain medications

Which data collection assessment would be performed to evaluate the effectiveness of furosemide administered to a client with congestive heart failure? Select all that apply. A. Daily weight B. Intake and output C. Monitor for edema D. Daily pulse oximetry E. Auscultate breath sounds

All of the above Rationale: Daily weight at the same time, on the same scale, and in the same clothing is important as it is an indication of fluid gains or losses. The nurse would also record daily intake and output and report intake exceeding output. The nurse would monitor for peripheral edema and document the findings. It is important to obtain and record vital signs and daily pulse oximetry as improving results relate to effectiveness of furosemide. The nurse would also auscultate breath sounds, look for jugular venous distension, and report abnormal data

Which assessment would the nurse perform before administering a dose of vancomycin to a client? Select all that apply. A. Creatinine B. Trough level C. Hearing ability D. Intravenous site E. Blood urea nitrogen

All of the above Rationale: Two major adverse effects of vancomycin are nephrotoxicity and ototoxicity. The nurse would assess the client's creatinine and blood urea nitrogen levels to determine renal function. The nurse would also assess the vancomycin trough levels to determine if the client's kidneys are clearing the medication. The nurse would assess for changes in hearing as a result of ototoxicity. Vancomycin can cause phelbitis, so the nurse would assess the intravenous site before initiating the infusion

Which activity would the nurse offer a 4-year-old child on bed rest? A. Give the child a stuffed animal B. Offer crayons with a coloring book C. Provide a set of large building blocks D. Take the child to the playroom between naps

B Rationale: Hand-eye coordination activities are age-appropriate for preschoolers and will allow for continued bed rest. Giving the child a stuffed animal may meet a security need but will not meet a 4-year-old child's developmental needs. Providing a set of large building blocks is inappropriate play for a child restricted to bed. The child is limited to bed rest, so taking the child to the playroom is not permissible

Which criterion is an indicator that the nitroglycerin sublingual tablets have lost their potency? A. Sublingual tingling is experienced B. The tablets are more than 3 months old C. The headache is less severe D. Onset of relief is delayed

B Rationale: Nitroglycerin tablets are affected by light, heat, and moisture. Loss of potency can occur after 3 months, reducing the medication's effectiveness in relieving pain. A new supply should be obtained routinely. Experiencing sublingual tingling indicates the tablets have retained their potency. Headaches may decrease over time; this is not an indicator of medication potency. A delay in relief reflects the ischemia, not the medication

When will the nurse monitor for a potential hypoglycemic reaction after administering daily regular insulin to a client with type 1 diabetes at 8 AM? A. At breakfast B. Before lunch C. Before dinner D. In the early afternoon

B Rationale: Regular insulin is short acting and peaks in 2 to 4 hours, which in this case will be at or before lunch. Breakfast is too soon; regular insulin peaks in 2 to 4 hours. Before dinner is too late; regular insulin peaks in 2 to 4 hours. The early after to too late; regular insulin peaks in 2 to 4 hours

Which nursing intervention would help an older adult experiencing urinary incontinence? Select all that apply. A. Provide nutritional support B. Provide voiding opportunities C. Avoid indwelling catheterization D. Provide beverages and snacks frequently E. Promote measures to prevent skin breakdown

B, D, E Rationale: An older adult should be provided voiding opportunities to minimize urinary incontinence. Indwelling catheterization should be avoided because this action increases the risk of infection, weakens bladder tone, and may cause discomfort. Measures to prevent skin breakdown should be take because the client may develop skin problems due to incontinence. Nutritional support and frequent beverages and snacks should be provided to a client with malnutrition

The registered nurse (RN) is caring for a client with severe diarrhea. Which task of the client care plan can be safely delegated to the unlicensed assistive personnel (UAP) by the RN? A. Administration of oral antidiarrheal B. Administration of intravenous antibiotics C. Administration of oral replacement fluids D. Administration of intravenous antiemetics

C Rationale: The scope of practice of the UAP is limited to perform basic care, feeding, and hygiene. Administering oral replacement fluids can be safely performed by the UAP, because it is a basic care that may also be included under feeding. Administering oral medications such as antidiarrheals can be safely delegated to a licensed practical nurse (LPN) or a licensed vocational nurse (LVN) according to guidelines but not to the UAP. Administering intravenous medication such as antibiotics or antiemetics is the role of the RN

Which recommendations would the nurse provide an overweight adolescent to promote weight loss? Select all that apply. A. Skip breakfast B. Sleep for long hours to reduce stress C. Perform physical activities regularly D. Eat small frequent meals throughout the day E. Reduce the intake of sugar and sweetened beverages

C, D, E Rationale: Regular physical activities can reduce risk factors associated with obesity and help a client lose weight. Eating small frequent meals throughout the day can increase a client's metabolism and help with weight reduction. Excess intake of sugar and sweetened beverages increases obesity. Sleeping for long hours increases the risk of obesity. Breakfast should not be skipped

After change-of-shift report, which client would the nurse assess first? A. A client with possible lung cancer who is scheduled for bronchoscopy B. A client with left pleural effusion who is scheduled for a thoracentesis C. A client with hospital-acquired pneumonia and decreased breath sounds D. A client with an acute asthma exacerbation and 85% oxygen saturation

D Rationale: A client with an oxygen saturation of 85% requires immediate assessment and interventions to improve oxygenation, such as administration of prescribed rapid-acting bronchodilators and administration of higher oxygen concentrations. There is no data indicating that the client who is schedules for bronchoscopy needs any urgent interventions. The client with a pleural effusion does not have symptoms that indicate any need for urgent assessment or interventions. Decreased breath sounds would be expected in a client with pneumonia and are not an indicator of a need for immediate assessment

A client is admitted to the hospital with gastrointestinal bleeding and subsequently has a nasogastric (NG) tube inserted. Prescriptions include irrigating the tube with normal saline as needed to maintain patency. The nurse assesses the NG tube and determines that it is not patent. Which initial action would the nurse take? A. Instill normal saline B. Assess breath sounds C. Auscultate for bowel sounds D. Check the tube for placement

D Rationale: Checking the tube for placement reduces the risk of introducing the irrigant into the lungs and is the initial action the nurse would take. Instilling normal saline increases the risk of introducing irrigant into the lungs if the tube is not in the stomach. Assessing for breath sounds is not related to the steps associated with instilling a nasogastric tube with an irrigant. Auscultating for bowel sounds is not related to the steps associated with instilling a nasogastric tube with an irrigant.

Which potential side effect of docusate sodium would a nurse include in discharge teaching of a client who had repair of an inguinal hernia? A. Rectal bleeding B. Fecal impaction C. Nausea and vomiting D. Mild abdominal cramping

D Rationale: Mild abdominal cramping is the only side effect of docusate sodium; this emollient laxative permits water and fatty substances to penetrate and mix with fecal material. Rectal bleeding is more likely to occur with a saline-osmotic laxative. Docusate sodium promotes defecation, not constipation. Nausea and vomiting are more likely to occur with a saline-osmotic laxative

Which blood pressure is optimal for an adolescent? A. 85/54 mmHg B. 95/65 mmHg C. 105/65 mmHg D. 110/65 mmHg

D Rationale: The optimal blood pressure of an adolescent is 110/65 mmHg. The average optimal blood pressure of an infant is 85/54 mmHg. The average optimal pressure of a toddler is 95/65 mmHg. The average optimal blood pressure seen in children between the ages of 6 and 13 years is 105/65 mmHg.

Which client statement indicates that the teaching about furosemide is understood? Select all that apply. A. "It may take 2 or 3 days for this medication to take effect." B. "I should wear dark glasses when outdoors during the day." C. "I should avoid lying flat in bed." D. "I need to change my position slowly." E. "I should eat more food that is high in potassium."

D, E Rationale: Furosemide may cause hypovolemia, which can result in orthostatic hypotension with sudden changes in position. With loop diuretics, such as furosemide, there is an increase in potassium excretion, so increased potassium intake is needed. The response to furosemide is rapid with diuresis beginning within an hour of administration. Furosemide does not cause photophobia, so dark glasses are not necessary. Lying flat in bed is unnecessary

Which member of the health care team is responsible for consulting other health care professionals in planning and delivering client care? A. Charge nurse B. Primary nurse C. Associate nurse D. Nurse navigator

B Rationale: The primary nurse is responsible for consulting other health care professionals in planning and delivering client care. The charge nurse delegates work to other health care team members in the absence of the delegator. The associate nurse is delegated to provide care to the client according to the primary nurse's specifications. The nurse navigator works through the health care system to help clients obtain high-quality care.

Which data would the nurse use to determine a client's score on the Braden Scale to predict a client's risk for developing pressure injuries? Select all that apply. A. Age B. Anorexia C. Hemiplegia D. History of diabetes E. Urinary incontinence

B, C, D, E Rationale: Anorexia causes nutritional problems; nutrition is a category on the Braden Scale. Hemiplegia causes mobility problems; this affects the categories of mobility, activity, and friction on the Braden Scale. Clients with a history of diabetes can also have peripheral neuropathy, causing numbness or loss of sensation in the hands and feet; sensory perception is a category on the Braden Scale. Urinary incontinence causes moisture, a category on the Braden Scale. Age is not used in the Braden Scale.

Which clinical finding in a client receiving morphine warrants immediate follow up by the nurse? Select all that apply. A. Polyuria B. Unconsciousness C. Bradycardia D. Dilated pupils E. Bradypnea

B, C, E Rationale: The central nervous system (CNS) depressant effect of morphine, if severe, can cause unconsciousness. The CNS depressant effect of morphine causes bradycardia and bradypnea. Morphine does not increase urine output. Morphine causes constriction of pupils

Which therapeutic outcome is expected after administering ibuprofen? Select all that apply. A. Diuresis B. Pain relief C. Temperature reduction D. Bronchodilation E. Anticoagulation F. Reduced inflammation

B, C, F Rationale: Prostaglandins accumulate at the site of an injury, causing pain; non-steroidal anti-inflammatory drugs (NSAIDS) like ibuprofen inhibit COX-1 and COX-2 (both are isoforms of the enzyme cyclooxygenase), which inhibit the production of prostaglandins, thereby contributing to analgesia. NSAIDs inhibit COX-2, which is associated with fever, thereby causing the temperature to decline. NSAIDs inhibition of COX-2 is associated with inflammation, thereby reducing inflammation. NSAIDS do not cause diuresis; reversible renal ischemia and renal insufficiency in clients with heart failure, cirrhosis, or hypovolemia can be potential adverse effects of NSAID. NSAIDs do not cause bronchodilation. Anticoagulation is an adverse effect, not a desired outcome; NSAIDs can impair platelet function by inhibiting thromboxane, an aggregating agent, resulting in bleeding

After the nurse teaches a client, who is obese, measures to calculate the body mass index, which client statement indicates effective learning? Select all that apply. A. "I should include sugared beverages in my diet." B. "I should lose at least half a pound to a pound each week." C. "My daily nutritional fat intake should be more than 30%." D. "I'll make sure to eat foods meeting my daily nutritional requirement E. "I should stay away from unhealthy foods between meals and after dinner."

B, D, E Rationale: A client's body mass index (BMI) height-weight range is appropriate when it is within 10% of the ideal body weight. To achieve this, the client should lose at least 0.5 to 1 lb. per week. The client's daily nutritional intake should meet the minimal dietary reference index. Refraining from eating unhealthy foods between meals and after dinner will help the client achieve an appropriate BMI. The client should avoid sugared beverages to achieve the appropriate BMI. Another effective way to achieve this is a daily fat intake less than 30% of total consumption

The nurse administers albuterol to a child with asthma. Which common side effect would the nurse monitor for in the child? A. Flushing B. Dyspnea C. Tachycardia D. Hypotension

C Rationale: Albuterol produces sympathetic nervous system side effects such as tachycardia and hypertension. Pallor, not flushing is a common side effect. Dyspnea is not a common side effect; this medication is given to decrease respiratory difficulty. Hypertension, not hypotension, is a common side effect

Which antidiarrheal medication would the nurse anticipate administering to a client with severe diarrhea who is prescribed intravenous fluids, sodium bicarbonate, and an antidiarrheal medication? A. Psyllium B. Bisacodyl C. Loperamide D. Docusate sodium

C Rationale: Loperamide inhibits peristalsis and prolongs transit time by its effect on the nerves in the muscle wall of the intestines. Psyllium in not an antidiarrheal; it is a bulk laxative that promotes easier expulsion of feces. Bisacodyl is a laxative, not an antidiarrheal; it increases gastrointestinal motility. Docusate sodium corrects constipation, not diarrhea; water and fat are increased in the intestine, permitting easier expulsion of feces

Which action would the nurse take when administering tetracycline? A. Administer the medication with meals or a snack B. Provide orange or other citrus fruit juice with the medication C. Administer the medication at least an hour before ingestion of milk products D. Offer antacids 30 minutes after administration if gastrointestinal side effects occur

C Rationale: Any product containing aluminum, magnesium, or calcium ions should not be taken in the hour before or after an oral dose of tetracyclines (with the expectation of doxycycline) because it decreases absorption by as much as 25% to 50%. Food interferes with absorption; it should be given 1 hour before or 2 hours after meals. Citrus juice does not improve absorption. Antacids will interfere with absorption

Which action would be most important for the nurse take when caring for a client who started furosemide 2 days ago and has a serum potassium level of 2.8 mEq/L? A. Hold the morning dose of the diuretic B. Continue to monitor the level to ensure that it stays within the normal limits C. Notify the primary health care provider of the critically low result D. Anticipate a prescription for an increase in the dosage of furosemide

C Rationale: The health care provider should be notified because a potassium level of 2.8 mEq/L is low. Normal range for serum potassium is 3.5 to 5 mEq/L. Clients who are on diuretics require monitoring of serum electrolytes, especially potassium and sodium, because they also are excreted with water. The nurse should not hold the diuretic or repeat the laboratory test unless advised by the health care provider. The client's serum potassium level is critically below the normal limit, and the health care provider should be notified. An increase in furosemide would cause an increased loss in potassium

After presenting information about fall risk assessments to nursing staff, which participant's statement needs review for corrective action? A. "We will assess every admission to the unit." B. "We will implement a valid falls risk assessment tool." C. "We will apply yellow wrist bands to high-risk clients." D. "We will use the admission fall assessment for the entire stay."

D Rationale: Falls risk assessment should be updated routinely through discharge, because fall risk may change throughout the hospital stay. Falls risk assessment should be completed on every client with a valid falls risk tool. High-risk fall clients should be identified.

A client is receiving intermittent nasogastric (NG) tube feedings to supplement oral nutritional intake. The nurse administers the feeding over 30 to 60 minutes to reduce which hazard? A. Distention B. Flatulence C. Indigestion D. Regurgitation

D Rationale: Because the cardiac sphincter of the stomach is slightly opened to admit the nasogastric tube, rapid instillation may result in regurgitation. Distention is diminished by avoiding the instillation of air with the feeding. The speed of a feeding does not cause flatulence but the administration of air may. Administering this feeding over 30 to 60 minutes may or may not decrease indigestion

Which amount of ceftriaxone 75 mg/kg/day divided bid would the nurse administer in one dose for a pediatric client who weighs 22 pounds? A. 375 mg B. 750 mg C. 825 mg D. 1650 mg

A Rationale: One dose of ceftriaxone contains 375 mg of medication. Twenty-two pounds is equal to 10 kg; 75 mg per kg is 750 mg total/day, divided into two doses would be 375 mg each

The nurse notes an older adult's admission orders include gentamicin for the treatment of osteomyelitis. Which laboratory report would the nurse review before beginning the medication? Select all that apply. A. Blood urea nitrogen (BUN) and creatinine B. Electrolytes and urinalysis C. Erythrocyte count D. Blood platelet count E. Serum thyroxin levels

A, B Rationale: Because gentamicin can increase the risk of nephrotoxicity, the nurse should assess a client's renal function before starting therapy. Dehydration can further increase the risk of nephrotoxicity; therefore, the client's hydration status should also be checked before starting therapy. Gentamicin generally does not affect erythrocyte and blood platelet counts nor does it affect serum thyroxin levels

The nurse plans to help a client get out of bed for the first time after surgery. Which assessment would the nurse complete before having the client sit on the side of the bed? Select all that apply. A. Presence of safe footwear B. Status of comfort and pain C. Appearance of wound and skin D. Patency of urinary catheter E. Observation of walking gait

A, B, C, D Rationale: Assessment of safe footwear before beginning the mobilization process will prevent the need for leaving the client to find footwear during the process. Pain and comfort have to be assessed before beginning to move the client. Wound assessment and catheter assessment are important to assure that the proposed activity will not lead to local injury or dislodgment of equipment from the activity. The client's walking gait cannot be observed until the client is out of bed.

Which intervention is beneficial to an older adult in acute care who is at risk of skin breakdown? Select all that apply. A. Providing thorough skin care B. Reducing shear forces and friction C. Providing beverages and snacks frequently D. Using a support surface base all the time E. Avoiding pressure with proper positioning

A, B, E Rationale: Providing an older adult with meticulous skin care may reduce the risk of skin breakdown. Reducing shear forces and friction prevents the development of pressure injuries. Pressure can be avoided with proper positioning. Beverages and snacks frequently provided to clients who are hospitalized due to dehydration. A supportive surface base is used based on risk factors

Which instructions would the nurse include when providing teaching to the parent of a child prescribed diuretics? Select all that apply. A. Fluid intake should be adequate B. Diuretics should be taken on an empty stomach C. Diuretics should be taken at the same time each day D. Diuretics may interfere with normal laboratory test values E. Sun or heat exposure should be avoided because of the risks of skin darkening

A, C, D Rationale: The parents should ensure that their child has adequate fluid intake to prevent dehydration. The medication should be taken every day at the same time to facilitate maximum therapeutic action. The parents should be informed that diuretics may interfere with normal laboratory test values such as serum levels of sodium, potassium, magnesium, and chloride. Diuretics should be taken with food or milk to prevent gastric irritation. Sun and heat exposure may cause fluid loss and heat stroke.

According to Maslow's hierarchy of needs, in which order would the nurse prioritize actions?

1. Providing a warm bath to the client to promote a good night's sleep 2. Encouraging the client to talk about fears and feelings 3. Assisting the client in getting out of bed to join family members for meals 4. Praising the client for administering insulin accurately Rationale: According to Maslow's hierarchy of needs, the basic needs of the client should be addressed first before moving toward the highest level of need. Thus providing a warm bath to promote a good night's sleep addresses the physiological (basic) need of the client. The next level of need is the safety and security need. The nurse encourages the client to talk about fears and feelings to meet this need. The next level is the love and belonging need. The nurse assists the client in getting out of bed and joining family members for meals to meet this need. Finally, the nurse addresses the self-esteem need by praising the client for administering insulin accurately

The nurse provides home care instructions to a client who was treated for injuries due to a fall. Which statement made by the client indicates a need for additional instruction? A. "I should walk on soft scatter rugs at home." B. "I should drink 3000 mL of water every day." C. "I should eat fruits and vegetables six times a day." D. "I should exercise the joints above and below the cast daily."

A Rationale: A client with injuries due to a fall must avoid having throw or scattered rugs at home to reduce the incidence of falls. The nurse would encourage the client to drink 3000 mL of water per day to promote optimal bladder and bowel function. The client would eat six small meals with foods rich in fiber, such as fruits and vegetables, to prevent constipation. The nurse would encourage the client to perform exercise above and below the cast daily for a speedy recovery

Which needs would the nurse address using Maslow's hierarchy of needs when caring for an emancipated minor reporting improper nutrition? A. Physiological B. Self- actualization C. Safety and security D. Love and belonging

A Rationale: According to Maslow's hierarchy of needs, the basic human needs are food, water, safety, and love. In the given scenario, the client reports improper nutrition, so the nurse would address the physiological need of the client first. Self-actualization deals with the need to achieve one's highest potential. Safety and security includes the physical and psychological safety of the client. Love and belonging includes the need to give and receive love and affection

Which substance would a client taking acetaminophen routinely be advised to avoid? A. Alcohol B. Caffeine C. Diphenhydramine D. Ibuprofen

A Rationale: Acetaminophen and alcohol are both hepatotoxic substances. Metabolites of acetaminophen, along with alcohol, can cause irreversible liver damage. Caffeine stimulates the cardiovascular system, not the liver. In addition, caffeine does not interact with acetaminophen. Diphenhydramine may be taken with acetaminophen. Ibuprofen may be taken with acetaminophen

The nurse administers acetaminophen to a child who complains of pain after abdominal surgery. The parent asks the nurse why her child isn't being given ibuprofen. Which response by the nurse is most appropriate? A. "It could prolong bleeding time." B. "It's contraindicated for young children." C. "It can suppress the healing of the incision." D. "It becomes ineffective when given for long periods."

A Rationale: Acetaminophen is not associated with bleeding complications like ibuprofen is, but if used long term, it can result in liver toxicity. Ibuprofen, a non-steroidal anti-inflammatory drug (NSAID) prolongs bleeding time; in the postoperative period, medications that interfere with clotting and prolong bleeding are contraindicated. Ibuprofen is safe for young children when administered in appropriate doses. Ibuprofen exerts an anti-inflammatory action; it does not interfere with the healing process. Tolerance for ibuprofen does not develop

For which physical assessment situation would the nurse use an alcohol-based hand sanitizer for hand hygiene? A. Before and after palpating a pulse B. Assessing a client with norovirus C. If the hand brushes a seeping dressing D. When the hands have contacted sputum

A Rationale: Alcohol-based hand sanitizer is appropriate when the hands are not visibly soiled or have not contacted any body fluid, so use of alcohol-based hand sanitizer before and after palpating a pulse is appropriate. Norovirus is not killed by alcohol-based hand sanitizer, so hand hygiene should be followed with soap and water. A seeping dressing and sputum involve contact with body fluid, so hand hygiene should be followed with soap and water

A client with a chronic obstructive pulmonary disease (COPD) exacerbation is receiving oxygen at 2 L/min per nasal cannula and has an oxygen saturation of 88%. Which action would the nurse anticipate taking next? A. Increasing oxygen flow rate to 3 L/min B. Preparing for intubation and assisted ventilation C. Administration of an inhaled rapid-acting bronchodilator D. Continuing to monitor the client with no therapy change

A Rationale: Because the client's oxygen saturation indicates hypoxemia, a higher flow rate of oxygen is needed. The nurse will continue to monitor oxygen saturation and respiratory rate and depth, because some (but not all) clients with COPD will have a decrease in respiratory drive when oxygen saturation is in the 95% to 100% range. Intubation and assisted ventilation is not indicated now, although it may be needed if higher oxygen flows fail to improve the client's oxygen saturation. Inhaled bronchodilators help open airways and are frequently used for clients with COPD exacerbation, but do not directly increase oxygen saturation. The nurse will continue to monitor the client, but a change in treatment is indicated because the client is hypoxemic

Which instruction would the nurse include in a health practices teaching plan for a female client with a history of recurrent urinary tract infections? A. 'Wear cotton underwear or lingerie.' B. 'Void at least every 6 hours around the clock.' C. 'Increase foods containing alkaline ash in the diet.' D. 'Wipe the perineum from back to front after toileting.'

A Rationale: Cotton allows air to circulate and does not retain moisture the way synthetic fabrics do; microorganisms multiply in warm, moist environments. Voiding frequently helps flush ascending microorganisms from the bladder, thereby reducing the risk for urinary tract infections; holding urine for more than 6 hours can lead to urinary tract infections. Foods high in acid, not alkaline ash, help acidify urine; this urine is less likely to support bacterial growth. Alkaline urine promotes bacterial growth. Wiping the genitals from back to front after toileting may transfer bacteria from the perianal area toward the urinary meatus, which will increase the risk for urinary tract infection

Which is the best nursing intervention to support a hospitalized child's nutrition who is apathetic toward eating? A. Asking the parent to visit at mealtimes B. Having a nursing assistant feed the child C. Providing diversional activity at mealtimes D. Eliminating the child's between-meal snacks

A Rationale: Dinner is frequently a family activity. Having a parent visit during meals may provide the child with additional emotional, social, and physical support, resulting in improved nutritional intake. The child will be resentful if fed by a staff member. Providing diversional activity at mealtimes may further inhibit the child's nutritional intake. Eliminating the child's between-meal snacks may not influence the child's overall intake; snacks may be preferred and will provide a source of nutrition

Which physiological alteration will the nurse monitor to best determine the effectiveness of a client's hydrochlorothiazide therapy? A. Blood pressure B. Decreasing edema C. Serum potassium level D. Urine specific gravity

A Rationale: Diuretics promote urinary excretion, which reduces the volume of fluid in the intravascular compartment, thus lowering blood pressure. The measure of blood pressure is the best determination of effectiveness because it is a direct measure of the desired outcome. A reduction in edema reflects effectiveness; however, multiple physiological processes, including venous competence, gravity, and disuse, maintain a significant degree of edema even when the diuretic is optimally effective. A lowered potassium level would indirectly indicate that the medication is working; however, this does not provide a good measure of effectiveness. Although specific gravity decreases with increased urinary output and thus would demonstrate that the medication is working, it is not a direct measure of the desired outcome. A measure of the reduction in intravascular pressure is preferable

Which play activity would the nurse provide for a 4-year-old child on bed rest? A. Finger painting on blank sheets of paper B. Providing a paint-by-number set C. Engaging in a checkers game with an adult D. Playing dominoes with a school-age roommate

A Rationale: Finger painting is appropriate for a child of this age; it provides the child an opportunity for free expression, and its freeform nature can give the child a sense of mobility. Painting by number within the lines of pictures requires more fine motor skills than most 4-year-olds possess; also, this activity does not allow freedom of expression or movement. Checkers is a game with too many rules for a 4-year-old to comprehend. Playing dominoes requires the ability to count and conserve numbers, which is beyond the ability of most 4-year-olds

When the nurse manager is evaluating the care of a client receiving oxygen through a nasal cannula, which finding indicates a need for more staff education about oxygen therapy? A. Oxygen flow rate is set to 8 L/min B. There is bubbling present in the humidifier C. Pressure areas of tubing along the ears are padded D. Smoking and open flame prohibited signs are clearly posted

A Rationale: Flow rates for nasal cannulas should be set no higher than 6 L/min, higher flows do not lead to an increase in FiO2, and high oxygen flow increases drying and irritation of mucous membranes. Bubbling of oxygen through the humidifier indicates that humidification of oxygen is occurring. Padding of pressure areas on the oxygen tubing decreases skin irritation and breakdown. Smoking and open flame prohibition signs are posted in places where oxygen is used to decrease the risk for fire

Which action would the nurse take when using a mechanical lift device to transfer a client? A. Fold the client's arms across the chest B. Place the top of the sling below the client's scapulae C. Wait until a prescription is written to use a mechanical lift D. Lift the sling at least 12 inches above the mattress

A Rationale: Folding the arms across the chest maintains both arms in a safe position during the transfer. During a safe transfer, the sling should extend from above the scapulae to the knees to provide appropriate support. The use of a mechanical lift is an independent function of the nurse. Raising the lift so that the sling is at least 12 inches above the mattress height is unsafe; during the transfer, the sling should be raised just high enough to clear the mattress

Which diuretic would the nurse anticipate administering to a client admitted with acute pulmonary edema? A. Furosemide B. Chlorothiazide C. Spironolactone D. Acetazolamide

A Rationale: Furosemide acts on the loop of Henle by increasing the excretion of chloride and sodium; is available for intravenous administration; and is more effective than chlorothiazide, spironolactone, and acetazolamide. Although it is used in the treatment of edema and hypertension, chlorothiazide is not as efficacious as furosemide. Spironolactone is a potassium-sparing diuretic; it is less efficacious than thiazide diuretics. Acetazolamide is used in the treatment of glaucoma to lower intraocular pressure

Which instruction would the nurse provide a client needing to collect a clean-catch urine specimen? A. 'Urinate a small amount, stop flow, and then fill one half of the specimen.' B. 'Collect a sample of the last urine voided during the night.' C. 'If anticipating a delay in delivery, keep the urine sample in a warm, dry area.' D. 'Send the urine sample to the laboratory within 6 hours of collection.'

A Rationale: Instruct the client to always collect midstream urine to send as a test specimen. Instruct the client to cleanse the perineum with the wipe provided, urinate a small amount, and then stop the flow. The client should then position the specimen cup a few inches from the urethra and resume urination, filling at least one half of the cup with urine. Instruct the client to collect the first sample voided in the morning because the urine is highly concentrated in the morning. Keeping the urine sample in the refrigerator helps reduce bacterial growth due to the urine's alkaline environment. The cells in the urine sample begin to break down in alkalinity; therefore instruct the client to send the sample to the laboratory as soon as possible after the collection

Which sign or symptom supports the nurse's suspicion that a client has overflow incontinence? A. Constant dribbling of urine B. Abrupt and strong urge to void C. Loss of urine with physical exertion D. Large amount of urine loss with each occurrence

A Rationale: Overflow incontinence is characterized by an involuntary loss of urine due to over-distention of the bladder when the bladder's capacity reaches the maximum. This condition is characterized by bladder distention up to the level of the umbilicus and constant urine dribbling. An abrupt and strong urge to void is a clinical manifestation of urge incontinence. Stress incontinence is characterized by loss of urine with physical exertion. Urge incontinence is characterized by the loss of large amounts of urine with each occurrence.

Oxygen given to clients during stage 4 of chronic obstructive pulmonary disease (COPD) would be administered in which manner? A. 1 to 2 L via nasal cannula to keep SaO2 above 90% B. 1 to 2 L via nasal cannula to maintain SaO2 at or above 95% C. 3 L via mask to maintain SaO2 at 95% D. Do not give oxygen because it may suppress hypoxic drive in client

A Rationale: Oxygen therapy usually is delayed until stage 4, which is very severe COPD. Usually it is administered at 1 to 2 L per minute to maintain SaO2 at or above 90%. 1 to 2 L to maintain the SaO2 above 95% is not necessary. Oxygen administration may not be necessary. 3 L of oxygen via a mask is unnecessary, and a level of 95% may suppress the hypoxic drive in client who are chronic CO2 retainers. Oxygen should not be given unless the chronic saturation level is less than 88%

Which response would a nurse give to a client with hypertension prescribed hydrochlorothiazide (HCTZ) who reports hearing that furosemide is more effective and requests a prescription change? A. "HCTZ has fewer side effects." B. "HCTZ does not cause dizziness." C. "HCTZ is only taken when needed." D. "HCTZ does not cause dehydration."

A Rationale: Side effects from thiazides generally are minor and rarely result in discontinuation of therapy. Dizziness is a side effect of all diuretics. All diuretic medications are taken regularly as directed. There is a potential for dehydration with all diuretics.

Which nursing professional has the maximum span of control? A. Nurse manager B. Registered nurse C. Unlicensed assistive personnel D. Licensed vocational nurse

A Rationale: Span of control, the number of people for whom one is ultimately responsible, increases with the number of individuals under a person's control. The nurse manager is the person responsible for the tasks performed by registered nurses, unlicensed assistive personnel, licensed vocational nurses/licensed practical nurses, and other nursing professionals; therefore the nurse manager has the maximum span of control

According to Quality and Safety Education for Nurses (QSEN), which phrase define patient-centered care? A. Understanding that the client is the source of control when providing care B. Functioning effectively within nursing and interprofessional teams to deliver quality care C. Using data to evaluate outcomes of care processes and designing methods to improve health care D. Minimizing the risk for harm to clients and health care workers through improved professional performance

A Rationale: The QSEN competency called patient-centered care requires the nurse to understand that the client is the source of control. The nurse would respect the values, beliefs, and preferences of the client to provide quality care. The QSEN competency called teamwork and collaboration states that the nurse would function effectively within nursing and interprofessional teams to provide quality care. Quality improvement involves using data to evaluate the outcomes of care processes and design methods to improve health care delivery system. Safety focuses on minimizing the risk for harm to clients and health care workers through improved professional performance

A client who had surgery for a resection of the colon and formation of a colostomy is scheduled to be discharged in several days. Which is a primary nursing intervention for this client? A. Determine the client's ability to care for the colostomy B. Show the client how to change the abdominal dressing C. Encourage the client to apply heat to the stoma opening D. Teach the client about special lifelong dietary precautions

A Rationale: The client's feelings, knowledge, and skills concerning caring for the colostomy must be assessed before discharge. Generally, the client no longer needs a dressing on the incision by the time of discharge. Heat is not applied to the stoma. After a colostomy, the client usually does not need a special diet; he or she usually is instructed to eat a regular diet and eliminate gas-producing foods

Which action would be appropriate to implement when collecting a 24-hour urine test? A. Start the time of the test after discarding the first voiding B. Discard the last voiding in the 24-hour period for the test C. Insert a urinary retention catheter to promote the collection or urine D. Strain the urine after each voiding before adding the urine to the container

A Rationale: The first voiding is discarded because that urine was in the bladder before the test began and should not be included. The last voiding should be placed in the specimen container because the urine was produced during the 24-hour time frame of the test. Discarding the last void in the 24-hour period for the test is not necessary; voided specimens are acceptable. Inserting a urinary retention catheter is not a standard step. Straining the urine after each voiding before adding the urine to the container is not necessary; this is done for clients with renal calculi

The nurse is teaching members of a health care team how to help disabled clients stand and transfer from the bed to a chair. Which step would the nurse do next after placing the nurse's arms under the client's axillae? A. Bending and then straightening their knees B. Bending at the waist and then straightening the back C. Placing one foot in front of the other and then leaning back D. Placing pressure against the client's axillae and then raising their arms

A Rationale: The leg bones and muscles are used for weight bearing and are the strongest in the body. Using the knees for leverage while lifting the client shifts the stress of the transfer to the caregiver's legs. By using the strong muscles of the legs, the back is protected from injury. Bending at the waist and then using the back for leverage is how many caregivers and people who must lift heavy objects sustain back injuries. The anatomical structure of the back is equipped only to bear the weight of the upper body. By leaning back, the client's weight is on the caregiver's arms, which are not equipped for heavy weight bearing. The caregiver's arms are not strong enough to lift the client. In the struggle to lift the client, the client and caregiver may be injured.

Which quality is the most important tool the nurse brings to the therapeutic nurse-client relationship? A. The self and a desire to help B. Knowledge of psychopathology C. Advanced communication skills D. Years of experience in psychiatric nursing

A Rationale: The nurse brings an understanding of self and basic principles of therapeutic communication; this is the unique aspect of the helping relationship. Knowledge of psychopathology, advanced communication skills, and years of experience in the field all support the psychotherapeutic management model and contribute to quality of care, but these are secondary to the offering of self and the fundamentals of good communication

Which available staff member would be best for the nursing manager to assign to take frequent vital signs for a client with suspected shock? A. Registered nurse B. Nursing student C. Licensed practical nurse D. Unlicensed assistive personnel

A Rationale: The rapid progression of shock requires the oversight of the registered nurse to assess and analyze vital signs. Nursing students have the education to take vital signs, but they do not have the experience or scope of practice to care for unstable clients. Licensed practical nurses have the education to obtain vital signs, but they do not have the scope of practice to care for unstable clients. Unlicensed assistive personnel are educated to accurately obtain vital signs, but they do not have the education or scope of practice to care for unstable clients

In the case of a client with which condition would it be most safe for the nurse to delegate the task of vital sign obtainment to an unlicensed assistive personnel (UAP)? A. Ankle sprain B. Possible pneumonia C. Substernal chest pain D. Unstable vital signs since admission

A Rationale: The task of checking the vital signs of the client with an ankle sprain, a non-urgent case, is safe to delegate to UAP. The nurse would not delegate checking vital signs of the client with possible pneumonia because that client would be an urgent status with possible respiratory compromise. The nurse would not delegate checking vital signs of the client with substernal chest pain because that client is an emergent case with possible hemodynamic compromise. The nurse would not delegate checking vital signs of the client with unstable vital signs because that is an emergent case that needs continuous assessment by the nurse

Nasogastric (NG) tube irrigations are prescribed for a client after abdominal surgery. The nurse instills 30 mL of saline solution, and 10 mL is returned. How would the nurse proceed? A. Record 20 mL as intake B. Increase the amount of solution C. Reposition the NG tube D. Irrigate the NG tube more frequently

A Rationale: This 20 mL (30 mL input - 10 mL output) must be accounted for in the intake and output. High suction may lead to adherence of mucosa to the tube and potential injury. Repositioning the nasogastric tube is unnecessary. Return of 10 mL indicates patency; more frequent irrigations are not indicated.

Which time would the nurse schedule the blood sample to be obtained when the primary health care provider prescribes trough levels of an antibiotic? A. Just before the medication is administered B. Between 30 and 60 minutes after the infusion is completed C. Six hours after the dose is completely infused D. In the morning before the client eats breakfast

A Rationale: Trough levels are measured in relation to the time a medication is administered. The trough level for a medication is drawn just before a medication is given, when the medication's level is at its lowest. Any other time would be inaccurate for a medication's trough level. The medication's peak level is drawn 30 to 60 minutes after the infusion is completed. Whether the client eats breakfast does not effect this medication's trough levels, because it is an intravenous infusion

Which action would the nurse take to avoid red man syndrome when preparing to administer a vancomycin infusion? A. Infuse slowly B. Change the intravenous (IV) site C. Reduce the dosage D. Administer vitamin K

A Rationale: Vancomycin should be infused to avoid the occurrence of the reaction known as 'red man syndrome.' Changing the IV site reduces the incidence of thrombophlebitis. Reducing the dosage is done in the setting of renal dysfunction. Administration of vitamin K is done to correct an elevated prothrombin time.

Which information will the nurse include when teaching a client with hypertension about metoprolol? A. Do not abruptly discontinue the medication B. Consume alcoholic beverages in moderation C. Report a heart rate of less than 70 beats per minute D. Increase the medication dosage if chest pain occurs

A Rationale: Abrupt discontinuation of metoprolol may cause rebound hypertension and an acute myocardial infarction. Alcohol is contraindicated for clients taking beta-adrenergic blockers such as metoprolol. The pulse rate can go lower than 70 beats per minute as long as the client is asymptomatic. Clients should never increase medication without medical direction

Which outcome would the nurse use to determine the effectiveness of sublingual nitroglycerin? A. Relief of anginal pain B. Improved cardiac output C. Decreased blood pressure D. Ease in respiratory effort

A Rationale: Cardiac nitrates relates smooth muscles of the coronary arteries; they dilate and deliver more blood to heart muscle, relieving ischemic pain. Although cardiac output may improve because of improved oxygenation of the myocardium, improved cardiac output is not a basis for evaluating the effectiveness of sublingual nitroglycerin. Although dilation of blood vessels and a subsequent drop in blood pressure is a reason why intravenous (IV) nitroglycerin may be administered, decreased blood pressure is not the basis for evaluating the effectiveness of sublingual nitroglycerin, which is indicated for pain relief. Ease in respiratory effort is not the basis for evaluating the medication's effectiveness

Which conclusion would the nurse make about the assessment finding of a client's very pale-yellow-colored urine? A. Dilute urine B. Hematuria C. Concentrated urine D. Myoglobinuria

A Rationale: Dilute urine tends to appear very pale-yellow in color. Dark-red or brown color urine indicates hematuria, the presence of blood in the urine. Dark-amber color urine indicates concentrated urine. Red color urine may indicate the presence of myoglobinuria

Which is an appropriate nursing action when caring for a client taking benazepril for hypertension? A. Assess for dizziness B. Assess for dark, tarry stools C. Administer the medication after meals D. Monitor the electroencephalogram

A Rationale: Dizziness may occur during the first few weeks of therapy until the client adapts physiologically to the medication. Dark, tarry stools are not a side effect of benazepril. Administering the medication after meals is unnecessary; however, if nausea occurs, the medication may be taken with food or at bedtime. The blood pressure should be monitored before and after administration. An EEG is unnecessary. Cardiac monitoring may be instituted because of possible dysrhythmias

The nurse gives the client with a history of constipation a list of foods to help prevent it. Which statement made by the client indicates further teaching is needed? A. 'I should eat eggs.' B. 'I should eat beans.' C. 'I should eat fresh fruits.' D. 'I should eat steamed vegetables.'

A Rationale: Eggs do no contain roughage and will not prevent constipation. Beans contain both soluble and insoluble fibers that promote intestinal peristalsis, preventing constipation. Raw fruits and steamed vegetables contain roughage that promotes intestinal peristalsis, preventing constipation

Which adverse effect of heparin would the nurse assess a client for during pregnancy? A. Osteoporosis B. Pelvic inflammatory disease C. Abnormal uterine contractions D. Suppression of uterine contractions

A Rationale: Heparin is safe to a fetus but may cause osteoporosis in a pregnant woman. Pelvic inflammatory disease and abnormal or suppressed uterine contractions are not associated with heparin

A client develops a deep vein thrombophlebitis in her leg 3 weeks after giving birth and is admitted for anticoagulant therapy. The nurse would anticipate developing a teaching plan for which anticoagulant? A. Heparin B. Warfarin C. Clopidogrel D. Enoxaparin

A Rationale: Heparin is the medication of choice during the acute phase of a deep vein thrombosis; it prevents conversion of fibrinogen to fibrin and of prothrombin to thrombin. Warfarin, a long-acting oral anticoagulant, is started after the acute stage has subsided; it is continued for 2 to 3 months. Clopidogrel is a platelet aggregate inhibitor and is used to reduce the risk of a brain attack. A low-molecular-weight heparin (e.g., enoxaparin) is not administered during the acute stage; it may be administered later to prevent future deep vein thromboses

Which reason would the nurse provide to a client who asks why blood tests are necessary after a health care provider prescribes peak and trough levels after initiation of intravenous antibiotic? A. "They determine if the dosage of the medication is adequate." B. "They detect if you are having an allergic reaction to the medication." C. "The tests permit blood culture specimens to be obtained when the medication is at its lowest level." D. "These allow comparison of your fever to changes in the antibiotic level."

A Rationale: Medication dose and frequency are adjusted according to peak and trough levels to enhance efficacy by maintaining therapeutic levels. Peak and trough levels reveal nothing about allergic reactions. Blood cultures are obtained when the client spikes a temperature; they are not related to peak and trough levels of an antibiotic. A sustained decrease in fever is the desired outcome, not a reduction just at peak serum levels of the medication

The nurse would include which instruction to the parents of a child being treated with oral ampicillin for otitis media? A. Complete the entire course of antibiotic therapy B. Herbal fever remedies are highly discouraged C. Administer the medication with meals D. Stop the antibiotic therapy when the child no longer has a fever

A Rationale: One antibiotic therapy is initiated, the antibiotics start to destroy specific bacterial infections that the health care provider is trying to treat. Antibiotic therapy takes a specific dose and number of days to completely eliminate the bacteria. If the caregivers start a dose and stop it before the course is complete, the remaining bacteria have a change to grow again, become resistant to antibiotic treatment, and multiply. The nurse would not discourage use of herbal fever remedies; however, the herbal treatment would be reviewed to see if it is contraindicated. Ampicillin would be taken 1 to 2 hours after meals. Antibiotic therapy would be completed as prescribed

A parent complains that his child's teeth have yellowed. The nurse understands that with prolonged use, which medication may be responsible? A. Tetracycline B. Promethazine C. Chloramphenicol D. Fluoroquinolones

A Rationale: When administered to neonates and infants, tetracycline may cause staining of developing teeth. Promethazine can cause respiratory depression in children under 2 years of age. Chloramphenicol can cause Gray baby syndrome, and fluoroquinolone may cause tendon rupture in pediatric clients

Which instruction regarding the use of fluticasone nasal spray is appropriate for client teaching? Select all that apply. A. Use the medication on a regular basis, not PRN. B. Clear the nasal passages before using the medication C. Discontinue use of the medication if nasal infection develops D. Remember that driving may be dangerous because of the medication's sedative effect E. Begin taking the medication 2 weeks before pollen season starts, and use it throughout the season

A, B, C Rationale: Fluticasone is a corticosteroid prescribed as a nasal spray in cases of sinusitis and rhinitis. The nurse would instruct the client to use the medication on a regular basis, not as needed, to clear the nasal passages before using the medication, and to discontinue use of the medication if nasal infection develops. Warning the client that driving may be dangerous because of the sedative effect would be beneficial for a client prescribed an antihistamine such as brompheniramine. The nurse should tell a client prescribed a mast cell stabilizer such as cromolyn spray to begin the medication 2 weeks before pollen season starts and to use it throughout the pollen season

Which interventions would the nurse manager include in a fall prevention program to decrease the number of falls on the unit? Select all that apply. A. Apply fall wristband B. Install bed safety alarms C. Establish a toileting schedule D. Allow client to ambulate to bathroom E. Use restraints to prevent the client from leaving the bed

A, B, C Rationale: The nurse would incorporate actions into the plan of care to decrease the risk of falls. These include applying a fall risk identification bracelet to the wrist. This alerts staff members which clients are at high risk for falls. The nurse would also use bed safety alarms as these are activated when clients are attempting to get out of bed without assistance. Establishing a toileting schedule can prevent falls from the client rushing to the bathroom. The client should not be allowed to walk to the bathroom without assistance. Restraints should be the last resort for confused and combative clients

Which clinical finding is commonly associated with hyperglycemia? Select all that apply. A. Polyuria B. Polydipsia C. Polyphagia D. Polyphrasia E. Polydysplasia

A, B, C Rationale: Polyuria is excessive urination with osmotic diuresis. Polydipsia is excessive thirst associated with hyperglycemia; thirst is the response to osmotic diuresis and glycosuria. Polyphagia is associated with the catabolic state induced by insulin deficiency. Polyphrasia is excessive talking associated with mental illness, not hyperglycemia. Polydysplasia is related to multiple developmental abnormalities and is unrelated to hyperglycemia.

Which information would the nurse include when teaching a client with coronary artery disease about aspirin therapy? Select all that apply. A. Take aspirin with food B. Report ringing in the ears C. Monitor and report excessive bleeding and bruising D. Do not mix aspirin with medications for erectile dysfunction E. Avoid OTC pain medications that contain aspirin

A, B, C, E Rationale: Aspirin is taken with food to avoid gastrointestinal upset. Ringing in the ears should be reported, because it is a sign of aspirin toxicity. Excessive bruising and bleeding should be reported, because these may be cause by decreased platelet aggregation caused by aspirin. OTC medications must be checked for aspirin. Nitrates, not aspirin, must be avoided by those takin erectile dysfunction medications because of the risk of a dangerous drop in blood pressure

Which strategies will promote safety and quality of client care on the unit? Select all that apply. A. Communicate with clarity and precision when designing multidisciplinary plans of care B. Create a safety huddle so all health care professionals are aware of the clinical objectives C. Emphasize electronic communication is quick and most effective means of sharing information in all situations D. Conduct communication simulations to increase knowledge about expertise of other health care disciplines E. Explain effective communication will take more time and effort compared with ineffective communication

A, B, D Rationale: Client care is becoming increasingly complex. High-quality, safe, and effective care calls for high-functioning health care teams. Communication is key. To improve quality and safety on the client care unit, team members should be encourage to communicate with clarity and precision when designing multidisciplinary plans of care. Creating a staff huddle ensures all staff members are aware of the clinical objectives of the day. Practicing communication in simulation can be useful to increase knowledge about the expertise of other health care disciplines on the health care team. Recognizing that electronic communication may be quick but, in some situations, may not be effective. For example, when client care issues are at stake, a focused, well-organized interdisciplinary meeting is more effective than a series of 'round-robin' emails. It usually takes about the same amount of time to communicate and collaborate ineffectively as it does to do it effectively.

When learning about the rules of leaders, which action of the nurse indicates effective implementation of the rules? Select all that apply. A. Communicating in a simple language with followers B. Giving an opportunity to the followers to express their views C. Instructing followers to decide the action to be performed D. Avoiding communicating clear boundaries with followers E. Ensuring proper eye contact while communicating with followers

A, B, E Rationale: An effective leader would use the simplest and fastest method to communicate with followers. An effective leader would give a change to the followers to express their views, because followers have many questions to ask. Proper eye contact ensures effective communication; therefore, an effective leader would maintain proper eye contact while communicating with followers. An effective leader would use critical thinking to decide the actions to be performed but should not instruct the followers. An effective leader would communicate clear boundaries with followers.

Which purposes are accomplished by applying the four elements of safety, critical thinking, stability, and time in the delegation of tasks? Select all that apply. A. Encouraging effective delegation decisions B. Ensuring effectiveness in performing elements of care C. Determining the level of readiness according to Hersey D. Developing the expectations of individuals to accomplish safety E. Assessing the ability of unlicensed assistive personnel (UAP) to perform the task

A, B, E Rationale: Applying the four elements of safety, critical thinking, stability, and time creates an integrative process that encourages effective delegation decisions. Elements of implementation would be understood to ensure effectiveness in performing elements of care. These elements must be considered while assessing the ability of the UAP to perform the task. To determine the readiness of a delegate, factors such as ability and willingness would be assessed according to Hersey's model of leadership. Practice acts, position descriptions, and policies are the elements required to form the expectations for which the individuals in certain conditions are expected to be able to accomplish safety

Which medication increases the risk for upper gastrointestinal (GI) bleeding? Select all that apply. A. Aspirin B. Ibuprofen C. Ciprofloxacin D. Acetaminophen E. Methylprednisolone

A, B, E Rationale: Non-steroidal anti-inflammatory drugs (NSAIDs), including acetylsalicylic acid and ibuprofen, and corticosteroids such as methylprednisolone are known cases of medication-induced gastrointestinal (GI) bleeding by causing irritation and erosion of the gastric mucosal barrier. Ciprofloxacin, an antibiotic, has not been associated with GI bleeding. Acetaminophen is a safe alternative to NSAIDs to reduce the risk of GI bleeding

The nurse identifies which anticoagulant medications as safe to administer during pregnancy for treatment of thrombophlebitis? Select all that apply. A. Heparin B. Warfarin C. Enoxaparin D. Clopidogrel E. Acetylsalicylic acid

A, C Rationale: Heparin may be used during pregnancy because it does not cross the placental barrier and will not cause hemorrhage in the fetus. Enoxaparin does not cross the placental barrier (formerly classified for pregnancy as category B). Warfarin crosses the placental barrier, causing hemorrhage in the fetus. Clopidogrel is a platelet aggregating inhibitor. It is not used for thrombophlebitis; it is used to reduce the risk of brain attack, transient ischemic attack, unstable angina, and myocardial infarction. Acetylsalicylic acid is a platelet aggregation inhibitor and is not recommended during pregnancy (formerly classified as category D.)

Which nursing interventions would provide safe oxygen therapy? Select all that apply. A. Check tubing for kinks B. Run wires under carpeting C. Post 'no smoking' signs in the client's rooms D. Place oxygen tanks flat in the carts when not in use E. Make sure the client is using oil-based products to lubricate the nose

A, C Rationale: Oxygen tubing should be checked for kinks during oxygen use. 'No smoking' signs should be posted in the client's rooms. Wires should not be kept under carpeting because heat buildup or friction can cause a fire. Oxygen tanks should be placed in an upright position in their carts or flat on floors. The client should use water-based lubricants when using oxygen therapy due to flammability of oil-based products

Which action would be included in an organization's policy for hand hygiene? Select all that apply. A. Wash hands before applying sterile gloves B. Wash hands before touching any of the client's personal items C. Wash with either soap and water or alcohol-based hand rub (ABHR) before client contact D. Wash with soap and water when hands are visibly soiled with blood E. Wash with ABHR if hands are not visibly soiled F. Wash hands, between fingers, and under nails for 60 seconds

A, C, D, E Rationale: A hand hygiene policy should include washing hands before applying sterile gloves, washing with either soap and water or an ABHR before client contact, washing with soap and water when hands are soiled with blood, and washing with an ABHR if hands are not visibly soiled. The hands should be washed after contact with inanimate objects in the immediate vicinity of the client. The Centers for Disease Control recommends washing the hands, between fingers, and under nails for a minimum of 20 seconds

Which benefit would be provided by administering patient-controlled analgesia (PCA) to a client after surgery? Select all that apply. A. Client is able to self-administer pain-relieving medications as necessary B. Amount of medication received is determined entirely by the client C. Decreases client dependency D. Relieves the nurse of monitoring the client E. Increases client sense of autonomy

A, C, E Rationale: The purpose of patient-controlled analgesia is to give the client the ability to self-administer pain-relieving medications as necessary; usually smaller amounts of analgesics are used with self-administration. The amount and dosage of the medication are programmed to prevent accidents or abuse. Medication levels are kept in a maintenance range, and pain relief is achieves without extreme fluctuations. The client isn't dependent on the nurse availability to administer medication. This increases the client's sense of autonomy. The nurse is not absolved of responsibility when PCA is used; monitoring the client for effectiveness, refilling the apparatus with the prescribed narcotic, and charting the amount administered and the client's response are required

Which example mentioned by the nurse belongs to the fifth level of Maslow's hierarchy of needs? Select all that apply. A. "A client wishes to become the best swimmer in the whole world." B. "A client tells the nurse that he or she has the urge to urinate every 10 minutes." C. "A client feels like a failure for not being able to complete college education." D. "A client tells the nurse that he or she wishes to become a successful engineer despite having dyslexia." E. "A client who is bedridden requests a physical therapist consultation to achieve maximum mobility."

A, D, E Rationale: According to Maslow's hierarchy of needs, the fifth level of needs include self-actualization needs. At this level, the individual wishes to reach his or her maximum potential while coping realistically with the situations of life. A client who wants to become the best swimmer in the world, become a successful engineer, or achieve maximum mobility after being bedridden is displaying self-actualization needs. Feeling the urge to urinate every 10 minutes is a physiological health problem that belongs to the first level of needs. A client who feels like a failure for not being able to complete college education may have a lack of self-worth and achievement. This situation belongs to the fourth level of needs.

Which suggestion would the nurse provide to the parent whose child has been constipated for 3 days? Select all that apply. A. Give laxatives to the child B. Reduce the child's fluid intake C. Include dairy products in the child's diet daily D. Increase the child's physical activity E. Include food with a high fiber content in the child's diet

A, D, E Rationale: Constipation is the infrequent and difficult passage of stools, but it can be manage by following certain measures. Laxatives may help ease the passage of stools to relieve constipation. Bowel movements can also be promoted by increasing physical activity and adding fiber to the diet to add bulk to the stool to relieve constipation. Low fluid intake and consumption of dairy products can increase the risk for constipation

Which suggestion would the nurse provide to the parent whose child has been constipated for 3 days? Select all that apply. A. Give laxatives to the child B. Reduce the child's fluid intake C. Include dairy products in the child's diet daily D. Increase the child's physical activity E. Include food with a high fiber content in the child's diet

A, D, E Rationale: Constipation is the infrequent and difficult passage of stools, but it can be managed by following certain measures. Laxative may help ease the passage of stools to relieve constipation. Bowel movements can also be promoted by increasing physical activity and adding fiber to the diet to add bulk to the stool to relieve constipation. Low fluid intake and consumption of dairy products can increase the risk for constipation

Which statement made by the nurse will be most significant when teaching strategies to reduce the risk for developing antibiotic-resistant infections? A. "Wash your hands frequently with warm soapy water." B. "Do not skip any prescribed doses of your antibiotics." C. "Do not save unfinished antibiotics for later use." D. "Do not stop taking the antibiotics when you feel better."

B Rationale: "Do not skip any doses of your antibiotics" is an umbrella answer to the question. The other statements are true, but they are not the most significant or inclusive of all the statements. If clients do not skip any doses of the antibiotic, they are unable to save any doses for later use, and are not able to stop taking the antibiotics when they feel better. Antibiotic-resistant infection develops when the hardiest bacteria survive and multiply. This may happen when a client stops taking an entire course of antibiotics, which leads to infections resistant to many antibiotics. A client should not skip any dose of an antibiotic. Hand washing is required to prevent infection, but there is not a relationship between hand washing and antibiotic-resistant infections. Taking the unfinished antibiotics at a later time for another infection is dangerous and may prove fatal if the antibiotics are outdated. Antibiotics should not be stopped, even if the client has started feeling better. The clients should take the full course of treatment. Noncompliance in taking the full course of prescribed antibiotics can lead to an antibiotic-resistant infection

The post-anesthesia care unit (PACU) receives a client post-abdominal surgery with a nasogastric tube with low-intermittent wall suction in place. Which initial action would the nurse implement when the client vomits 90 mL of bile-colored fluid? A. Elevate the head of the bed B. Check the patency of the tube C. Administer the prescribed antiemetic D. Encourage the client to take several deep breaths

B Rationale: A nasogastric tube attached to suction removes gastric secretions and prevents vomiting. However, if the tube obstructs, secretions accumulate, leading to distention, nausea, and vomiting. Initially, turn the client on her or his side to prevent aspiration. Administration of an antiemetic may occur after verification of tube patency. Deep breathing will not prevent vomiting if a nasogastric tube is not patent

Wrist restraints have been prescribed for a confused client to keep the client from pulling out a nasogastric tube and indwelling urinary retention catheter. Which nursing intervention is correct regarding restraints? A. Have the prescription renewed every 48 hours B. Assess the client's condition per hospital protocol C. Provide range-of-motion exercises to the client's wrists every shift D. Document output from the nasogastric tube and catheter every 2 hours

B Rationale: A restraint impedes the movement of a client; therefore a client's condition needs to be assessed every hour. All restraints are required to be prescribed every 24 hours. Restraints should be removed and activity and skin care provided at least every 2 hours to prevent contractures and skin breakdown. Output from tubes may be monitored hourly, but generally does not need to be documented as frequently as every 2 hours. Generally, output from tubes is emptied, measured, and documented at the end of each shift

Which nursing action illustrates the Quality and Safety Education for Nurses (QSEN) competency called informatics? A. Ensure that the risk of harm to clients and health care workers is decreased by improving professional performance B. Use information and technology to communicate, manage knowledge, mitigate error, and support decision-making C. Integrate best current evidence with clinical expertise and client preferences and values to deliver quality health care D. Use data to monitor the outcomes of health care processes and implement improvement methods to design and test changes to improve quality of health care

B Rationale: According to informatics, the nurse would use information and technology to communicate, manage knowledge, mitigate errors, and support decision-making. According to the QSEN competency called safety, the nurse would ensure that the risk of harm to clients and health care workers is decreased by improving professional performance. The QSEN competency called evidence-based practice states that the nurse would integrate best current evidence with clinical expertise along with client preferences and values to deliver quality health care. The QSEN competency called quality improvement states that the nurse would use data to monitor the outcomes of health care processes and implement improvement methods to design and test changes to improve quality of health care.

The nurse teaches a client about wearing thigh-high anti-embolism elastic stockings. Which instruction would be correct to include? A. 'You do not need to wear them while you are awake, but it is important to wear them at night.' B. 'You will need to apply them in the morning before you lower your legs from the bed to the floor.' C. 'If they bother you, you can roll them down to your knees while you are resting or sitting down.' D. 'You can apply them either in the morning or at bedtime, but only after the legs are lowered to the floor.'

B Rationale: Applying antiembolism elastic stockings in the morning before the legs are lowered to the floor prevents excessive blood from collecting and being trapped in the lower extremities as a result of the force of gravity. Elastic stockings are worn to prevent the formation of emboli and thrombi, especially in clients who have had surgery or who have limited mobility, by applying constant compression. It is contraindicated for antiembolism elastic stockings to be applied and worn at night, rolled down, or applied after the legs are lowered to the floor

Which action would be taken first when a staff nurse is instructed by the charge nurse to give the scheduled warfarin dose to a client whose current international normalized ration (INR) is 6? A. Refuse to give the unsafe medication dose B. Remind the charge nurse of the INR result C. Ask the hospital pharmacist to talk to the nurse manager D. Ask the health care provider whether to give the medication

B Rationale: Because the INR is at an unsafe level, warfarin would not be given. Professional communication would include first clarifying concerns with the charge nurse, by discussing the abnormal INR result and reasons for not administering another dose of warfarin. Although the nurse could refuse to give the medication as the first action, this is not likely to foster professional communication or workplace relationships. Direct communication with coworkers about concerns is more professional and fosters better relationships than having a third party (such as the pharmacist) addresses concerns. Because the INR is prolonged and the warfarin would be unsafe to give, the nurse does not need to ask the health care provider about giving the warfarin but would notify the provider about the INR result

Which statement is important for the nurse to teach a client prescribed psyllium 3.5 g twice a day for constipation? A. "Urine may be discolored." B. "Each dose should be taken with a full glass of water." C. "Use only when necessary because it can cause dependence." D. "Daily use may inhibit the absorption of some fat-soluble vitamins."

B Rationale: Because this medication has a strong affinity for fluids, it will swell in the intestine. The large bulk stimulates peristalsis. A full glass of fluid taken at the same time will help minimize the risk of esophageal obstruction or fecal impaction. Senna, a stimulant laxative, is among the safest laxatives on the market. It is useful with prolonged therapy because it is not systemically absorbed and is not potent in its action. Prolonged use of lubricant or saline/osmotic laxatives, not bulk-forming laxatives, can inhibit the absorption of some fat-soluble vitamins

Which urine characteristics is consistent with a urinary tract infection? A. Smoky B. Cloudy C. Orange-amber D. Yellow-brown

B Rationale: Cellular debris, white blood cells, bacteria, and pus can cause the urine to become cloudy. Dark, smoky urine usually suggests hematuria. Orange-amber color of urine may indicate concentrated urine; also, it can be cause by phenazopyridine or foods such as beets. Yellow-brown to dark color of urine indicates excessive bilirubin

The nurse is teaching the parent of a child prescribed a high dose of oral prednisone for asthma. Which information is critical for the nurse to include when teaching about this medication? A. It protects against infection B. It should be stopped gradually C. An early growth spurt may occur D. A moon-shaped face will develop

B Rationale: Gradual weaning from prednisone is necessary to prevent adrenal insufficiency or adrenal crisis. Prednisone depresses the immune system, thereby increasing susceptibility to infection. The medication usually suppresses growth. A moon face may occur, but it is not a critical, life-threatening side effect

Which insulin would the nurse conclude has the fastest onset of action? A. NPH insulin B. Insulin lispro C. Regular insulin D. Insuline glargine

B Rationale: Insulin lispro has an onset of 0.25 hours, a peak action of 0.5 to 1.5 hours, and a duration of 3 to 4 hours. Neutral protamine Hagedorn (NPH) or intermediate-acting insulin has an onset of 1.5 hours, a peak acting of 4 to 12 hours, and a duration of 18 to 24 hours. Regular insulin has an onset of 0.5 hours, a peak action of 1 to 5 hours, and a duration of 6 to 10 hours. Insulin glargine has an onset of 1 to 1.5 hours, no peak action, and a duration of 20 to 24 hours.

Which nursing action will best evaluate the effectiveness of the furosemide in managing a client with heart failure and acute pulmonary edema? A. Performing daily weights B. Auscultating breath sounds C. Monitoring intake and output D. Assessing for dependent edema

B Rationale: Maintain adequate gas exchange and minimizing hypoxia with pulmonary edema are critical; therefore assessing the effectiveness of furosemide therapy as it relates to the respiratory system is more important. Furosemide inhibits the reabsorption of sodium and chloride from the loop of Henle and distal renal tubule, causing diuresis; as diuresis occurs fluid moves out of the vascular compartment, thereby reducing pulmonary edema and the bilateral crackles. Although a liter of fluid weighs approximately 2.2 pounds and weight loss will reflect the amount of fluid lost, it will take time before a change in weight can be measured. Although identifying a greater output vs. intake indicates the effectiveness of furosemide, it is the client's pulmonary status that is most important with acute pulmonary edema. Although the lessening of a client's dependent edema reflects the effectiveness of furosemide therapy, it is the client's improving pulmonary status that is the best indicator of how furosemide improves the client's condition

A child with type 1 diabetes is receiving 15 units of regular insulin and 20 units of NPH insulin at 7:00 AM each day. Which time would the nurse anticipate a hypoglycemic reaction from the NPH insulin to occur? A. Before noon B. In the afternoon C. Within 30 minutes D. During the evening

B Rationale: NPH insulin is an intermediate-acting insulin that peaks approximately 6 to 8 hours after administration. It was administered at 7:00 AM, so between 1:00 PM and 3:00 PM is when the nurse would anticipate that a hypoglycemic reaction would occur. Noon is when a reaction from a short-acting insulin is expected. Short-acting insulin peaks 2 to 4 hours after administration. Within 30 minutes of administration is when a reaction from a rapid-acting insulin is expected. Rapid-acting insulin peaks 30 to 60 minutes after administration. During the evening or nighttime is when a reaction from a long-acting insulin is expected. Long-acting insulin has a small peak 10 to 16 hours after administration

Which purpose of metoclopramide administered intravenously 30 minutes before initiating chemotherapy for cancer of the colon would the nurse explain to the client? A. To stimulate production of gastrointestinal (GI) secretions B. To stimulate peristalsis of the upper gastrointestinal (GI) tract C. To prolong excretion of the chemotherapeutic medication D. To increase absorption of the chemotherapeutic medication

B Rationale: Prokinetic medications such as metoclopramide stimulate peristalsis in the GI tract. This enhances the emptying of stomach contents into the duodenum to decrease gastroesophageal reflux and vomiting, all of which are precipitated by chemotherapeutic agents. Metoclopramide does not stimulate the production of gastrointestinal secretions. Metoclopramide has no effect on the excretion of chemotherapeutic medications. Metoclopramide has no effect on the absorption of chemotherapeutic medications

Which intervention would the nurse implement when providing care for an older adult male client who is immobile and incontinent of urine? A. Restrict the client's fluid intake B. Regularly offer the client a urinal C. Apply incontinence pants D. Insert an indwelling urinary catheter

B Rationale: Regularly offering the urinal is the first step. Retraining the bladder includes a routine pattern of attempts to void, which may increase bladder muscle tone and produce a conditioned response. Restricting fluid intake can result in dehydration and a urinary tract infection in an older client. Applying incontinence pants does not address the cause of the incontinence, promotes skin breakdown, and may lower the client's self-esteem. Inserting an indwelling urinary catheter increases the risk of a urinary tract infection. Insertion of an indwelling urinary catheter requires a primary health care provider's prescription

Which therapeutic communication technique is a coping strategy to help the nurse and client adjust to stress? A. Sharing hope B. Sharing humor C. Sharing empathy D. Sharing observations

B Rationale: Sharing humor is a therapeutic communication technique that involves using a coping strategy that adds perspective and helps the nurse and client adjust to stress. Nurse would recognize that hope is essential for healing and communicate a sense of possibility. Sharing empathy is the ability to understand and accept another person's reality, accurately perceive feelings, and communicate this understanding to the other. Sharing observations often helps a client communication without the need for extensive questioning, focusing, or clarification

Which condition would be prevented by the addition of spironolactone to the medication regimen of a client with cirrhosis of the liver who has been taking chlorothiazide? A. Hyponatremia B. Hypokalemia C. Ascites D. Peripheral neuropathy

B Rationale: Spironolactone is a potassium-sparing diuretic often used in conjunction with thiazide diuretics. The provider was prompted to add spironolactone to the chlorothiazide to prevent potassium loss. It stimulates sodium excretion so will not prevent hyponatremia. Spironolactone is a relatively weak diuretic that will not have a significant effect on ascites. Peripheral neuropathy is not a concern in this scenario and spironolactone would not have an effect on it if it was a concern

The nurse is caring for a client with a new colostomy. Which client outcome is most important for achievement of long-range goals associated with adjusting to a new colostomy? A. Mastery of techniques of colostomy care B. Readiness to accept an altered body function C. Awareness of available community resources D. Knowledge of necessary dietary modifications

B Rationale: The client must be ready to accept changes in body image and function; this acceptance will facilitate mastery of the techniques of colostomy care and optimal use of community resources. Specific knowledge can be imparted only when an individual is ready to learn; it requires acceptance of a new body image.

The nurse observes that an older adult client seldom eats the meat on the meal trays. The nurse discusses this observation with the client, and the client states, 'I only eat meat once a week because old people don't need protein every day.' Which need would the nurse address in her or his reply? A. Need for home-delivered meals B. Foods that meet basic nutritional needs C. Effect of aging on the need for some foods D. Need for meat at least once per day throughout life

B Rationale: The need for nutrients, including protein, that meet basic nutritional needs continues throughout life. The priority is to educate the client, although home-delivered meals may be one way to provide adequate nutrition. Aging has no effect on the specific nutrients needed; however, it may influence digestion or absorption of food. Protein is needed every day, but it does not have to be in the form of meat

Which clinical indicator would the nurse monitor to determine if the client's simvastatin is effective? A. Heart rate B. Triglycerides C. Blood pressure D. International normalized ratio (INR)

B Rationale: Therapeutic effects of simvastatin include decreased levels of serum triglycerides, low-density lipoprotein (LDL), and cholesterol. Heart rate and blood pressure are not related to simvastatin. INR is not related to simvastatin; it is a measure used to evaluate blood coagulation

A client has a nasogastric feeding tube inserted, and the health care provider prescribes the feeding to be instituted immediately. Which action would the nurse take first? A. Instill normal saline into the tube to maintain patency B. Obtain an x-ray to verify that the tube is in the stomach C. Auscultate with epigastric area while instilling 30 mL of air D. Withdraw stomach contents to observe color and consistency

B Rationale: X-ray verification of tube placement is required before anything is instilled into the nasogastric tube. Administering a feeding through a misplaced tube can cause the formula to enter the client's lungs. Instilling normal saline into the tube to maintain patency is unsafe. The normal saline will enter the client's lungs if the tube is in the wrong place. Auscultating the epigastric area while instilling 30 mL of air and withdrawing a small amount of stomach contents to verify tube placement are not definitive ways to ensure correct placement of the nasogastric tube. Once placement is verified by an x-ray, these methods may be used before initiating a feeding

The nurse is preparing to initiate antibiotic therapy for a client who developed an incisional infection. Which task would the nurse ensure has been completed before starting the first dose of intravenous antibiotics? A. Red blood cell count B. Wound culture C. Knee x-ray D. Urinalysis

B Rationale: A wound culture always should be completed before the first dose of antibiotic. A wound culture is obtained to determine the organism that is growing. A broad spectrum antibiotic often is given first; after the organism has been identified an organism-specific antibiotic can be given. There is no indication that a red blood cell count is needed; however, a white blood cell count would be beneficial. A urinalysis is not needed, because data gathered during the assessment indicate an incisional infection. At the early stage of the infection, there is no need to obtain a knee x-ray

A child is receiving 45 units of intermediate-acting insulin at 7:00 AM and 7:00 PM. Which statement by the nurse is most appropriate when discussing bedtime snacks with the parent? A. "Offer a snack at bedtime if there are signs of hyperglycemia." B. "Provide a bedtime snack to prevent hypoglycemia during the night." C. "Withhold the snack after dinner to prevent hyperglycemia during sleep." D. "Leave a snack at the bedside in case the child becomes hungry during the night."

B Rationale: Intermediate-acting insulin peaks in 4 to 12 hours; a bedtime snack will prevent hypoglycemia during the night. Offering a snack at bedtime if there are signs of hyperglycemia is unsafe because it will intensify the hyperglycemia; if hyperglycemia is present, the child needs insulin. Bedtime snacks are recommended for people taking intermediate-acting insulin. When hypoglycemia develops, the child will be asleep; the snack should be eaten before bed

Which concept would the nurse consider when caring for school-aged children who are obese? A. Enjoyment of specific foods is inherited B. There are familial influences on childhood eating habits C. Childhood obesity is usually not a predictor of adult obesity D. Children with obese parents are destined to become obese themselves

B Rationale: Studies have demonstrated that culture and family eating habits have an effect on a child's eating habits. Inheritance is not known to influence eating habits, although it is believed that other hereditary factors are associated with obesity. Childhood obesity is a known predictor of adult obesity. Children with obese parents are not necessarily destine to become obese themselves

Which response would a nurse give to a client diagnosed with type 1 diabetes who states, "I hate shots. Why can't I take the insulin in tablet form?" ? A. "Your diabetic condition is too serious for oral insulin." B. "Insulin is poorly absorbed orally, so it is not available in a tablet." C. "Insulin by mouth causes a high incidence of allergic and adverse reactions" D. "Once your diabetes is controlled, your primary health care provider might consider oral insulin."

B Rationale: The chemical structure of insulin is altered by gastric secretions, rendering it ineffective. There is no such thing as oral insulin; this comment about the seriousness of the diabetic condition may increase anxiety. There are no data to support the statement regarding allergic or adverse reactions, and insulin is given parenterally, not orally. Insulin is not absorbed but is destroyed by gastric secretions; there is no insulin that is effective if taken by mouth

Which action by the nurse increase safety in the clinical setting? Select all that apply. A. Wear a mask when performing daily hygiene B. Remain current with evidence and best practices C. Use only quality resources, especially websites D. Report faulty equipment as soon as discovered E. Sterilize all intravenous pumps between clients

B, C, D Rationale: Actions by the nurse that increase safety in the clinical setting include keeping current evidence and best practices, using only quality resources when looking up information, and reporting faulty equipment immediately and not using it until the repairs are completed. Wearing a mask when performing hygiene is not necessary unless the client is in droplet, airborne, or neutropenic precautions. Intravenous pumps should be cleaned with a high-level disinfections, not sterilization, process

Which responses would the nurse expect a client experiencing hypoglycemia to exhibit? Select all that apply. A. Nausea B. Palpitations C. Tachycardia D. Nervousness E. Warm, dry skin F. Increased respirations

B, C, D Rationale: Palpitations are of neurogenic origin associated with hypoglycemia; the sympathetic nervous system is stimulated by the decline in blood glucose. Tachycardia occurs with low serum glucose levels because of sympathetic nervous system activity. Nervousness, anxiety, and shakiness occur as a result of sympathetic nervous system stimulation associated with hypoglycemia. Nausea, vomiting, and abdominal cramps are associated with hyperglycemia. Warm, dry skin is a sign of hyperglycemia, caused by dehydration associated with osmotic diuresis related to glycosuria. The skin will be cool and moist with hypoglycemia. Increased respirations are signs of hyperglycemia and diabetic ketoacidosis from insufficient insulin to prevent fat breakdown for energy. There is no change in respirations with hypoglycemia.

Which activities would the nurse perform to meet the client's safety and security needs based on Maslow's hierarchy of needs? Select all that apply. A. Providing a cold bath to reduce the client's body temperature B. Positioning the bed in a low position and keeping the side rails up C. Monitoring vital signs, such as blood pressure to decrease the risk of falls D. Observing a client who has suicidal tendencies to prevent adverse incidents E. Collaborating with family members to provide emotional support for the client post-surgery

B, C, D Rationale: As per Maslow's hierarchy of needs, to meet the safety and security needs of the client, the nurse would position the bed in a low position and keep the side rails up to provide physical safety for the client. Monitoring vital signs, including blood pressure, to prevent risk for falls is an example of a nursing activity that meets the client's safety and security needs. Observing a client who has suicidal tendencies to prevent adverse incidents is an example of a nursing activity that meets the client's safety and security needs. Providing a cold bath to the client to reduce body temperature is an example of a nursing activity that meets the client's physiological needs. Collaborating with family members to provide emotional support to the client post-surgery is an example of a nursing activity that meets the client's love and belonging needs.

Which instructions would the nurse give to an older adult with decreased perception of touch? Select all that apply. A. 'Use a cane for support when walking.' B. 'Hold on to handrails while ambulating.' C. 'Look where your feet are placed while walking.' D. 'Wear shoes that give good support while walking.' E. 'If you are unable to change your position frequently, request assistance.'

B, C, D, E Rationale: Decreased perception of touch is a physiological change of the nervous system associated with aging. The client may experience decreased sensory perception that may cause the client to fall. Holding onto the handrails for directional guidance and support may help allow time for careful foot placement. The client should be instructed to look carefully where feet are placed while walking to prevent falling. Good support from wearing shoes while walking may reduce the risk of falling in clients with decreased perception of touch. If the client is unable to change position frequently, the caretake should change the position of the client every hour while the client is in bed or in a chair. A cane may be useful for a client with a musculoskeletal condition or injury but will not help with decreased sensory perception

The nurse recognizes that premature infants are at risk for increased sensitivity to medications due to which factors? Select all that apply. A. Body size B. Blood-brain barrier C. Renal drug excretion D. Hepatic drug metabolism E. Protein binding of the drugs

B, C, D, E Rationale: In infants, immaturity of the pharmacokinetic processes, including the blood-brain barrier, renal drug excretion, hepatic drug metabolism, and protein binding of the drugs, as well as drug absorption, all contribute to increased sensitivity to drugs. Body size does not contribute to the infant's increased sensitivity to pharmacological agents

Which data would the nurse use to determine a client's score on the Braden Scale to predict a client's risk for developing pressure injuries? Select all that apply. A. Age B. Anorexia C. Hemiplegia D. History of diabetes E. Urinary incontinence

B, C, D, E Rationale: Anorexia causes nutritional problems; nutrition is a category on the Braden Scale. Hemiplegia causes mobility problems; this affects the categories of mobility, activity, and friction on the Braden Scale. Clients with a history of diabetes can also have peripheral neuropathy, causing numbness or loss of sensation in the hands and feet; sensory perception is a category on the Braden Scale. Urinary incontinence causes moisture, a category on the Braden Scale. Age is not used in the Braden Scale.

The newly promoted registered nurse (RN) is about to start the process of delegation for the first time in their career. Which experiences foster professional self-confidence in the early career stage for an RN? Select all that apply. A. Inability to comprehend B. Experience in clinical delegation C. Application of theory to practice D. Development of clinical judgement E. Engagement with a nursing mentor

B, C, D, E Rationale: Clinical delegation experiences foster professional self-confidence in the early career of an RN. The nursing practicum experiences of applying theory to practice and developing clinical judgement will also advance the RN's ability to become a successful delegator and foster professional self-confidence. Engagement with a nursing mentor fosters professional self-confidence in the early career of an RN. The RN must have the ability to comprehend the legal authority of delegation decisions

Which statement(s) related to initial assessment of blood pressure by the nurse require(s) correction? Select all that apply. A. 'Deflating the cuff too slowly will show false high diastolic reading.' B. 'The stethoscope applied too firmly against the antecubital fossa will show a low systolic reading.' C. 'If the blood pressure in the left arm is 110/80 mmHg and in the right arm it is 130/80 mmHg, it is reportable.' D. 'Having the client's arm unsupported while assessing blood pressure will result in a false low reading of blood pressure.' E. 'It is normal to have blood pressure of 110/80 mmHg in the left arm and blood pressure of 120/80 mmHg in the right arm.'

B, D Rationale: False low diastolic readings are obtained in clients if the stethoscope is held too firmly against the antecubital fossa. Keeping the arm unsupported while assessing blood pressure results in false high blood pressure values. False high diastolic readings are obtained when the cuff is deflated too slowly. Pressure differences greater than 10 mmHg between the two arms should be reported because it indicates vascular problems. Normally there is a difference of 5 to 10 mmHg of blood pressure between the arms

Which finding would the nurse expect when assessing a client hospitalized for dehydration? Select all that apply. A. Protruding eyeballs B. Postural hypotension C. Client reports eating an average of two meals daily D. Skin on forehead remains tented after being pinched E. Weight loss of 4 ounces over 4 days

B, D Rationale: Postural hypotension is an indicator of dehydration. To determine dehydration in the adult, the nurse should test for decreased skin turgor. To assess for dehydration, pinch the skin over a bone with little or no underlying fat, such as the sternum or forehead. If the skin remains tended after it is released, the client is dehydrated. The eyeballs may be sunken, not protruding, in the presence of dehydration. The client's report of eating two meals a day does not indicate dehydration. A weight loss of 4 ounces does not indicate dehydration

The nurse provides education to a client about how to prevent constipation. The nurse concludes that the teaching is understood when the client makes which statements? Select all that apply. A. 'I can eat potatoes at dinner daily.' B. 'I should drink at least six glasses of water every day.' C. 'I should eat eggs for breakfast three times a week.' D. 'I can include bran muffins in my breakfast daily.' E. 'I will walk every day as part of my exercise regimen.'

B, D, E Rationale: Six glasses of water a day keep the feces soft, which prevents constipation. Whole grains such as bran muffins are high in roughage, which prevents constipation. Walking increases intestinal motility, which helps prevent constipation. Potatoes and eggs do no contain roughage and will not prevent constipation

Which instruction will the nurse include in a teaching plan for a client taking a calcium channel blocker such as nifedipine? Select all that apply. A. Reduce calcium intake B. Report peripheral edema C. Expect temporary hair loss D. Avoid drinking grapefruit juice E. Change to a standing position slowly

B, D, E Rationale: Peripheral edema may occur as a result of heart failure and must be reported. Grapefruit juice affects the metabolism of calcium channel blockers and should be avoided. Changing positions slowly helps reduce orthostatic hypotension. Reducing calcium intake is unnecessary because calcium levels are not affected. Hair loss does not occur

The nurse delegates the tasks of caring for a postpartum client. During assessment, the nurse observes an infection in the client caused by lack of hygiene. Which member of the health care team is most likely responsible for the client's condition? Select all that apply. A. Primary health care provider B. Registered nurse (RN) C. Licensed practical nurse (LPN) D. Licensed vocational nurse (LVN) E. Unlicensed assistive personnel (UAP)

B, E Rationale: The RN may delegate the tasks of basic client care to the UAP, but is responsible and accountable for the outcome of the task. UAP are responsible for the client's basic hygiene and comfort. A primary health care provider may delegate the task of client care to the RN and other assistive personnel. An LPN or licensed vocational nurse (LVN) generally carries out tasks such as administering oral medication but is not typically responsible for client hygiene if UAPs are available.

A client has a surgically created colostomy. Which is the most effective nursing intervention initially to help the client accept the colostomy? A. Provide literature containing factual data about colostomies B. Ask a member of a support group to come to speak with the client C. Begin to teach self-care of the colostomy by introducing equipment D. List the names of important people who have had colostomies

C Rationale: Beginning with equipment is less threatening and may stimulate feelings of mastery. Providing literature containing factual data about colostomies may be helpful, but introducing needed equipment will be more likely to provide the client with initial acceptance. Asking a member of a support group to come to speak with the client is helpful but may take time and may not meet immediate needs. Listing important people who have had colostomies may be helpful but is not an initial step to take toward the client's acceptance of the colostomy

A client with emphysema reports increased shortness of breath and becoming increasingly anxious. The health care provider prescribes oxygen at 1 L/min via nasal cannula. The nurse recognizes that this prescription is appropriate for which reason? A. The client does not need any more than 1 L/min B. High concentrations of oxygen cause alveoli to rupture C. High concentrations of oxygen eliminate the respiratory drive in some patients D. The oxygen at 1 L/min should be enough to diminish the anxiety

C Rationale: Clients with emphysema are used to low levels of oxygen and high levels of carbon dioxide. Oxygen is the stimulus for breathing for these clients instead of the natural breathing stimulus. Too much oxygen will knock out the stimulus to breathe. High concentrations of oxygen will not cause a rupture. The client actually could need more oxygen; however, if a higher concentration is given, it will knock out the respiratory drive. The oxygen is being given because of the shortness of breath

Which mechanism of action explains how diuretics reduce blood pressure? A. They facilitate vasodilation B. They promote smooth muscle relaxation C. They reduce the circulating blood volume D. They block the sympathetic nervous system

C Rationale: Diuretics decrease blood volume by blocking sodium reabsorption in the renal tubules, thus promoting fluid loss and reducing arterial pressure. Direct relaxation of arteriolar smooth muscle is accomplished by vasodilators, not diuretics. Vasodilators, not diuretics, act on vascular smooth muscle. Medications that act on the nervous system, not diuretics, inhibit sympathetic vasoconstriction

Which represents appropriate management of a client's nasogastric (NG) tube in the immediate postoperative period after gastroduodenostomy? A. Advancing the tube to the original insertion depth if the tube becomes dislodged B. Obtaining a prescription to vigorously irrigate the nasogastric tube if clogging is noted C. Carefully monitoring the nasogastric tube to ensure that the tube is patent and the suction is working D. Reporting the presence of bright red gastric aspirant in the suction canister during the immediate postoperative period

C Rationale: Ensuring that the nasogastric tube is patent and that the suction is working properly are priorities for the postoperative client to prevent retention of gastric secretions that may lead to abdominal distention, nausea, vomiting, and further serious complications. Advancing the tube to the original insertion depth if the tube becomes dislodged is not recommended. Improper reinsertion may result in the aspiration of gastric contents. Vigorous irrigation of the nasogastric tube, even if clogged, is not recommended because this can cause damage to the gastric mucosa. Finally, the presence of bright red gastric aspirant in the suction canister for the first 24 hours after surgery is a normal finding in the postoperative period

A client with a recent colostomy expresses concern about the inability to control the passage of gas. Which recommendation would the nurse make? A. Eliminate foods high in cellulose B. Decrease fluid intake at mealtimes C. Avoid foods that in the past caused flatus D. Adhere to a bland diet before social events

C Rationale: In general, foods that caused flatus preoperatively will continue to do so after a colostomy. Foods high in fiber necessarily are not related to formation of flatus. Reducing foods high in fiber can result in constipation; a regular diet is encouraged. Decreasing fluid intake at mealtimes is not a factor in the formation of flatus. A bland diet may be used initially after the colostomy, but then the client should progress to a regular diet; to control the formation of flatus, the client should eliminate foods that produce gas

Which principle would the nurse consider when assisting a client with crutches to learn the four-point gait? A. Elbows should be kept in rigid extension B. Most of the client's weight should be supported by axillae C. The client must be able to bear weight on both legs D. The affected extremity should be kept off the ground

C Rationale: In the four-point gait, the client brings the left crutch forward first, followed by the right foot; then the right crutch is brought forward, followed by the left foot. Thus both legs must be able to bear some weight. Although the arms are extended to allow the hands to bear weight, the elbows are not maintained in this position. Pressure on the axillae may damage nerves in the area. Both extremities must be able to bear weight and touch the ground

While hospitalized, a client with diabetes is observed picking at calluses on the feet. Which intervention would the nurse implement immediately? A. Warn the client of the danger of infection B. Suggest that the client wear white cotton socks C. Teach the client the importance of effective foot care D. Check the client's shoes for their fit in the area of the calluses

C Rationale: Inadequate foot care can lead to skin breakdown, poor healing, and subsequent infection, so the nurse should teach the client the importance of effective foot care. Warning the client of the danger of infection can increase anxiety and reduce the client's ability to learn. Suggesting that the client wear white cotton socks is only one aspect of effective foot care; synthetic fibers that wick moisture are preferred. Although important, checking the client's shoes for their fit in the area of the calluses is not comprehensive foot care.

Which example is a one-on-one communication between the nurse and another person? A. Small-group communication B. Intrapersonal communication C. Interpersonal communication D. Transpersonal communication

C Rationale: Interpersonal communication is a one-on-one interaction between a nurse and another person that often occurs face-to-face. Small-group communication is interaction that occurs when a small number of people meet. Intrapersonal communication is a form of communication that occurs within an individual. Transpersonal communication is an interaction that occurs within a person's spiritual domain

A client with a coronary occlusion is experiencing chest pain and distress. Which is the primary reason that the nurse administers oxygen? A. To prevent dyspnea B. To prevent cyanosis C. To increase oxygen concentration to heart cells D. To increase oxygen tension in the circulating blood

C Rationale: Oxygen increases the transalveolar oxygen gradient, which improves the efficiency of the cardiopulmonary system. This enhances the oxygen supply to the heart. Increased oxygen to the heart cells will improve cardiac output, which may or may not prevent dyspnea. Pallor, not cyanosis, usually is associated with myocardial infarction. Although increasing oxygen tension in the circulating blood may be true, it is not specific to heart cells

A child recovering from a severe asthma attack is given oral prednisone 15 mg twice daily. Which intervention would be a priority for the nurse? A. Having the child rest as much as possible B. Checking the child's eosinophil count daily C. Preventing exposure of the child to infection D. Offering sips of water when administering the medication

C Rationale: Prednisone reduces the child's resistance to certain infectious processes and, as an anti-inflammatory medication, masks infection. The child will self-limit activity depending on respiratory status. The eosinophil count is often consistently increased in children with asthma. The child will need adequate hydration to help loosen and expel mucus

Which rationale supports the nurse's instruction that a client with chronic kidney disease is to avoid all salt substitutes? A. A person's body tends to retain fluid when a salt substitute is included in the diet B. Limiting salt substitutes in the diet prevents a buildup of waste products in the blood C. Salt substitutes contain potassium, which must be limited to prevent abnormal heartbeats D. The salt substitute substances interfere with capillary membrane transfer, resulting in anasarca

C Rationale: Salt substitutes usually contain potassium, which can lead to hyperkalemia; dysrhythmias are associated with hyperkalemia. Chronic kidney disease already places the client at a higher risk for hyperkalemia because of poor elimination of fluids and electrolytes. Sodium, not salt substitutes, in the diet causes retention of fluid. Salt substitutes do not contain substances that influence blood urea nitrogen and creatinine levels; these are the result of protein metabolism. There is not a substance in the salt substitutes that interferes with capillary membrane transfer. Anasarca is extensive fluid in the tissues throughout the body and more extensive than typical edema

Which condition would the nurse identify as the likely cause of profound weakness and nervousness in a client that became confused shortly after self-administering the morning dose of 10 units of regular insulin and 25 units of NPH insulin after a light breakfast with no additional intake in the 3 hours since that time? A. Hyperglycemia B. Hyperinsulinemia C. Hypoglycemia D. Hypoinsulinemia

C Rationale: Severe hypoglycemia is a finding in diabetic clients who take insulin and miss a meal. Signs and symptoms of hypoglycemia are nervousness, weakness, confusion, and disorientation. Hyperglycemia is rare in clients who are on insulin therapy and decrease their intake. Hyperinsulinemia is a condition where an excess of insulin is produced by the pancreas in response to conditions such as insulin resistance or insulinomas. Hypoinsulinemia refers to abnormally low levels of insulin in the blood

Which nursing action is most appropriate to help reduce the likelihood of an older adult client falling during the night? A. Moving the client's bedside table closer to the bed B. Encouraging the client to take and available sedative C. Instructing the client to call the nurse before going to the bathroom D. Assisting the client to telephone home to say goodnight to the spouse

C Rationale: Statistics indicate that the most frequent cause of falls by hospitalized clients is getting up or attempting to get up to go to the bathroom unassisted. Although moving the bedside table closer to the bed is helpful in reducing falls because it moves the bedside table closer to the client's center of gravity, it is not the primary intervention to prevent falls. Sedatives contribute to the risk for falls by altering the client's sensorial abilities. Although talking to the spouse may calm the client and contribute to sleep, it does not reduce the incidence of falls

Which physiological change occurs in older adults and warrants the nurse teaching the client about safety tips to prevent falls? A. Slowed movement B. Cartilage degeneration C. Decreased bone density D. Decreased range of motion (ROM)

C Rationale: Teaching safety tips to prevent falls would best help a client with decreased bone density. If a client experiences slow movements, the nurse should not rush the client because the client may become frustrated if hurried. Providing a client with cartilage degeneration with a moist heat source such as a shower or a warm compress is beneficial because this action may increase blood flow to the area. The nurse should assess a client's ability to perform activities of daily living and mobility to help improve the self-care skills of clients with a decreased range of motion

Which point regarding ethics and values requires further education? A. The nurse's point of view offers a unique voice in the resolution of ethical dilemmas B. Professional nursing promotes accountability, responsibility, advocacy, and confidentiality C. The American Nurses Association (ANA) code of ethics ensures that the code remains constant D. Standards ethics in health care consist of autonomy, beneficence, nonmaleficence, justice, and fidelity

C Rationale: The American Nurses Association (ANA) code of ethics reviews and revises the code regularly to reflect changes in practice. The nurse's point of view offers a unique voice in the resolution of ethical dilemmas by including knowledge based upon clinical and psychosocial observations. Professional nursing promotes accountability, responsibility, advocacy, and confidentiality. Standards ethics in health care consist of autonomy, beneficence, nonmaleficence, justice, and fidelity

Which statement by an obese adolescent about health dietary habits and the risk associated with obesity indicates the need for further counseling? A. "I should do exercise." B. "I should play more outdoor games." C. "I should watch more TV to reduce the stress." D. "I should modify my diet and have lots of vegetables and water."

C Rationale: The cause of obesity can be stress, but rather than watching TV to reduce stress, some other activities like dancing, which involve physical movements, can be done. Any type of physical exercise helps in fat burning. Playing outdoor games not only is a physical exercise but also helps reduce stress. Reducing the consumption of a fat-rich diet and replacing it with vegetable will reduce the amount of fat consumed by the adolescent, and drinking a high amount of water detoxify the body

A client has a nasogastric tube (NG) in place. Which health care team member is most likely to be assigned to provide medications to the client through the nasogastric tube? A. Charge nurse B. Nursing manager C. Licensed practical nurse D. Unlicensed assistive personnel

C Rationale: The licensed practical nurse (LPN) is suitable to provide medications to the client through the nasogastric tube because the LPN has sufficient knowledge to provide medications through the tube. The charge nurse may not be available to provide medications to the clients because the charge nurses have other responsibilities. Nursing managers do not provide medications; rather they delegate tasks to the registered nurses and other health care team members. Unlicensed assistive personnel are not eligible to provide medications to clients through the nasogastric tube

Which information is important for the nurse to include in the teaching plan of a client who will be taking furosemide and digoxin after discharge from the hospital? A. Maintenance of a low-potassium diet B. Avoidance of foods high in cholesterol C. Signs and symptoms of digoxin toxicity D. Importance of monitoring output

C Rationale: The risk of digoxin toxicity increases when the client is receiving digoxin and furosemide, a loop diuretic; loop diuretics can cause hypokalemia, which potentiates the effects of digoxin, leading to toxicity. Digoxin toxicity can result in dysrhythmias and death. When a client is receiving a loop diuretic, the diet should be high in potassium. Although teaching the need to avoid foods high in cholesterol may be included in the teaching plan, it is not the priority. It is not necessary to monitor output

Which assessment finding would the nurse report to the health care provider when immediate postoperative care to a client with a newly placed ostomy? A. Moderate edema of the stoma B. Excessive gas issuing from the stoma C. Blanching, dark red to purple color of stoma D. Small amount of blood oozing from the stoma

C Rationale: The stoma should be rosy pink to red in color. A blanching, dark red to purple stoma indicates inadequate blood supply to the stoma or bowel and should be reported to the health care provider. Mild to moderate edema is normal initially and will resolve over the first 6 weeks. Excessive gas is common for the first 2 weeks post-surgery. It is normal for a small amount of blood to ooze from the stoma when touched because of its high vascularity

Which information would the nurse include when teaching a type 2 diabetic controlled with oral anti-diabetic medications who is prescribed regular insulin when admitted for elective surgery? A. "You will need a higher serum glucose level while on bed rest." B. "The stress of surgery may cause hypoglycemia." C. "With insulin, dosage can be adjusted to your changing needs during recovery from surgery." D. "The possibility of surgical complication is greater when a client takes oral hypoglycemics."

C Rationale: There is better control of blood glucose with short-acting (regular) insulin. The level of glucose must be maintained as close to normal as possible; elevated glucose levels are not desirable for clients on bed rest. The stress of surgery will precipitate hyperglycemia (not hypoglycemia), which is best controlled with exogenous insulin. Oral hypoglycemics do not increase surgical complications

Which strategy would improve safety when the nurse manager institutes strategies to decrease the omission of important information during communication between staff nurses and health care providers? A. Require health care providers to print prescriptions instead of using cursive writing B. Use the "read-back" method when taking phone prescriptions from health care providers C. Employ SBAR (Situation, Background, Assessment, and Recommendation) communication D. Devise standing orders for the five most common admitting diagnoses on the client care unit

C Rationale: To improve communication and prevent omissions of important information between staff nurses and health care providers, the nurse manager could employ SBAR communication as a guiding framework. Handwritten prescriptions should be avoided, but, if the need presents, the health care provider should take care to write legibly, and the nurse should verify the understanding of the written prescription. Using the read-back method is helpful for telephone prescriptions if electronic medical records are not in use; however, it does not guarantee that there will be no omissions of information. Devising standing orders would not improve communication between the staff nurses and health care providers.

Which rationale would the nurse include to address the client's concern about why both warfarin and intravenous (IV) heparin are needed at the same time for a partial occlusion of the left common carotid artery? A. This permits the administration of smaller doses of each medication B. Giving both medications allows clot dissolution while preventing new clot formation C. Heparin provides anticoagulant effects until warafin reaches therapeutic levels D. Administration of heparin with warfarin provides immediate and maximum protection against clot formation

C Rationale: Warfarin is administered orally for 2 to 3 days to achieve the desired effect on the international normalized ration (INR) level before heparin is discontinued. Because each medication affects a different part of the coagulation mechanism, dosages must be adjusted separately. These medications do not dissolve clots already present. That this approach immediately provides maximum protection against clot formation is not the reason for the administration of both medications; warfarin will not exert an immediate therapeutic effect

When a client arrives in the emergency department and reports dyspnea, which finding by the nurse indicates than an inhaled bronchodilator may be needed for treatment? A. Oral temperature 101 degrees F B. Use of accessory muscles for breathing C. Inspiratory and expiratory wheezes D. Dullness to percussion at lung bases

C Rationale: Wheezing is caused by airway spasm and narrowing, and inhaled bronchodilators will relax bronchospasm and open the airways rapidly. Fever may indicate pneumonia and the nurse would anticipate obtaining cultures and administration of intravenous fluids and antibiotics. Use of accessory muscle is seen with many respiratory problems and inhaled bronchodilators may not be helpful, depending on the client diagnosis. Dullness to percussion may be associated with diagnoses such as pleural effusion and would not be treated with an inhaled bronchodilator

After completing a week of antibiotic therapy, an infant develops oral thrush. Which medication is indicated for treatment of this condition? A. Acyclovir B. Vidarabine C. Nystatin D. Fluconazole

C Rationale: White, adherent patches on the tongue, palate, and inner aspects of the infant's cheeks indicate oral candidiasis (thrush). Oral candidiasis is caused by a fungus called Candida albicans. Nystatin is an antifungal agent prescribed to treat oral thrush in an infant. Acyclovir and vidarabine are antiviral agents and are not used to treat oral candidiasis in the infant. Fluconazole can effectively treat oral thrush, but its use in infants is not approved by the US Food and Drug Administration

Which preventative would the nurse anticipate will be prescribed daily to avoid straining due to constipation for a client who has had a recent brain attack (cerebrovascular accident/stroke)? A. Stimulant laxatives such as bisacodyl B. Tap-water enemas C. Stool softener D. Saline laxatives such as magnesium citrate

C Rationale: A stool softener can soften stool and promote defecation, thus avoiding the Valsalva maneuver. Stimulant laxatives are not recommended for daily use because laxative dependency has occurred in some clients. Enemas may precipitate a forcible exhalation against a close glottis during evacuation. Elevated intraabdominal and intrathoracic pressures associated with the Valsalva maneuver increase intracranial pressure and should be avoided. Also, daily enemas promote dependence. Saline laxatives can cause hypermagnesemia if given this frequently.

When a client is seen in the emergency department with sudden onset severe dyspnea, coughing, and wheezes, which prescribed treatment would the nurse administer first? A. Inhaled corticosteroid B. Normal saline infusion C. Albuterol via nebulizer D. Intravenous methylprednisolone

C Rationale: The client symptoms suggest acute asthma attack or anaphylaxis. Inhaled bronchodilators like albuterol act within a few minutes to relax bronchospasm, decrease bronchiolar inflammation, and dilate bronchioles. Inhaled corticosteroids are not rapidly acting and can be given after inhaled bronchodilators. Normal saline would be needed, but the nurse would not wait to give the bronchodilator while infusing saline. Intravenous corticosteroids like methylprednisolone take several hours to be effective and would not be the priority treatment

Which would the nurse include in dietary teaching for a client with a colostomy? A. Liquids should be limited to 1 L per day B. Non-digestible fiber and fruits should be eliminated C. A formed stool is an indicator of constipation D. The diet should be adjusted to result in manageable stools

D Rationale: Each person will need to experiment with diet after a colostomy to determine what foods are best tolerated and produce stools that are manageable depending on the type of colostomy. Liquids are typically not limited unless there is a specific reason such as cardiac or renal disease. Foods high in fiber such as fruit should be included in the diet as tolerated. Depending on the type of colostomy and the diet, a formed stool is acceptable and does not indicate a constipating diet

A child is prescribed fluticasone after an acute asthma attack. Which instruction would the nurse give the family about the administration of this medication? A. 'Fluticasone needs to be taken with food or milk.' B. 'Fluticasone is primarily used to treat acute asthma attacks.' C. 'The child should such on hard candy to help relieve dry mouth.' D. 'Watch for white patches in the mouth and report to the health care provider.'

D Rationale: Fluticasone is a steroid commonly administered by way of inhalation for long-term control of asthma symptoms. Oral thrush is a side effect that manifests as while patches. Fluticasone is administered via inhalation so food or milk is not needed before administration. Dry mouth is not a side effect of fluticasone

Which client response during the insertion of a nasogastric tube indicates to the nurse that the client is experiencing serious difficulty with the insertion? A. Choking B. Redness C. Gagging D. Cyanosis

D Rationale: If the nasogastric tube is passed accidentally into the trachea rather than the esophagus, it will occlude the airway, causing cyanosis; this is a serious problem that must be correct immediately. Choking may occur as the tube passes through the back of the throat; this commonly occurs with insertion of a nasogastric tube and is a temporary problem. Facial flushing (floridity) may result if the client attempts to fight the passage of the tube; this commonly occurs with insertion of a nasogastric tube and is temporary problem. Gagging may occur as the tube passes from the nasal passage through the pharynx; this commonly occurs with insertion of a nasogastric tube and is a temporary problem

Which statement made by a client prescribed metformin extended release to control type 2 diabetes mellitus indicates the need for further education? A. "I will take the medication with food." B. "I must swallow my medication whole and not crush or chew it." C. "I will notify my doctor if I develop muscular or abdominal discomfort." D. "I will stop taking metformin for 24 hours before and after having a test involving dye."

D Rationale: Metformin must be withheld for 48 hours before the use of iodinated contrast materials to prevent lactic acidosis. Metformin is restarted when kidney function has returned to normal. Metformin is taken with food to avoid adverse gastrointestinal effects. If crushed or chewed, metformin XL will be released too rapidly and may lead to hypoglycemia. Muscular and abdominal discomfort is a potential sign of lactic acidosis and must be reported to the health care provider.

Which treatment would the nurse anticipate administering to a toddler who has acetaminophen poisoning? A. Oxygen B. Naloxone C. Amyl nitrate D. N-acetylcysteine

D Rationale: N-acetylcysteine is used to treat an acetaminophen poisoning. Oxygen is used to treat carbon monoxide poisoning. Naloxone is used to treat opioid overdoses. Amyl nitrate is used to treat cyanide poisoning.

A client is admitted to the hospital with a diagnosis of acute pancreatitis. The health care provider's prescriptions include nothing by mouth and total parenteral nutrition (TPN). The nurse explains that the TPN therapy provides which benefit? A. Is the easiest method for administering needed nutrition B. Is the safest method for meeting the client's nutritional requirements C. Will satisfy the client's hunger without the discomfort associated with eating D. Will meet the client's nutritional needs without causing the discomfort precipitated by eating

D Rationale: Providing nutrients by the intravenous route eliminates pancreatic stimulation, reducing the pain experienced with pancreatitis. TPN is one of the most complex methods for administering needed nutrition compared to enteral nutrition and oral intake. TPN creates many safety risks for the client. Hunger can be experienced with TPN therapy

Which safety consideration is the nurse following when obtaining the client's family history and checking for a medical alert bracelet? A. Client identification B. Injury prevention for staff C. Injury prevention for clients D. Risk for errors and adverse events

D Rationale: Risk for errors and an adverse event is a safety consideration applied by nurses to identify any possible risks to a client's health. It involves obtaining the client's family history and checking the client for a medical alert bracelet. Client identification involves providing an identification (ID) bracelet for each client and using two unique identifiers. Injury prevention for staff is a safety consideration that involves use of standard precautions at all times to prevent any violence involving clients. Injury prevention for clients is a safety consideration that involves keeping rails on the stretcher or placing it in a lower position

The nurse is teaching a client about safe insulin administration. Which statement made by the client indicates the need for further education? A. "I should see whether the insulin is expired." B. "I should keep a daily logbook of times of insulin injection." C. "I should keep my medication in its original labeled container." D. "I should administer insulin only if there are any symptoms."

D Rationale: The client should administer insulin regardless of symptoms and should maintain the dose and daily schedule as prescribed by the primary health care provider. Before administering insulin, the client should check the expiration date. The client should maintain a daily logbook of insulin injections. The client should keep the medication in its original labeled container and refrigerate if needed.

When the oxygen saturation of a client with pneumonia is at 89% to 90% while using a nonrebreather mask, which collaborative action would the nurse anticipate? A. Administration of oxygen using a simple face mask B. Use of a Venturi mask for administration of high-flow oxygen C. Continued oxygen administration with the nonrebreather mask D. Oxygen administration with bi-level positive airway pressure (BiPAP)

D Rationale: The client's oxygen saturation is low even with the fraction of inspired oxygen (FiO2) levels of up to 95% that can be administered with a nonrebreather mask, and another method of oxygen administration is needed. BiPAP adds positive pressure during inspiration to oxygen administration, decreasing some of the work of breathing and improving tidal volumes and gas exhange without some of the risks that are associated with intubation and mechanical ventilation. The FiO2 levels of 40% to 60% that can be delivered by a simple face mask are adequate to improve oxygenation for this client. A Venturi mask can provide FiO2 levels of up to 50%, which is less than the client is already receiving. A nonrebreather mask can deliver FiO2 levels of up to 95%, but this is not adequate for this client who has hypoxemia even with use of the nonrebreather mask

The nurse is teaching an adolescent with a sprained ankle how to use one crutch when walking. Which statement indicates that no further teaching is necessary? A. "I shouldn't use my crutch on stairs." B. "After a month I can stop using the crutch." C. "I should place the crutch in front of my bad ankle." D. "I should use the crutch on the side of my good ankle."

D Rationale: The crutch is positioned on the unaffected side and advanced with the affected leg; the crutch supports the body's weight while the client is walking on the affected leg. The crutch should be used on the stairs to provide a wide base and extra support when the client goes up or down the stairs. A sprained ankle should heal in less than 1 week, after which the crutch should no longer be needed. Positioning the crutch in front of the affected foot will place that foot in a weight-bearing position without support, defeating the purpose of the crutch

The nurse instructs a client to breathe deeply to open collapsed alveoli. Which explanation could the nurse offer to explain the relationship between alveoli and improved oxygenation? A. The alveoli need oxygen to live B. The alveoli have no direct effect on oxygenation C. Collapsed alveoli increase oxygen demand D. Oxygen is exchanged for carbon dioxide in the alveolar membrane

D Rationale: The exchange of oxygen and carbon dioxide occurs in the alveolar membrane. If the alveoli collapse, this exchange cannot occur because pulmonary ventilation is reduced. Explaining this process in simple terms to a client may increase compliance with recommended breathing exercises aimed at improving oxygenation. Alveoli do have a direct effect on oxygenation. The statements that alveoli need oxygen to live and that collapsed alveoli increase oxygen demand are nonspecific regarding the pathophysiology of the alveolar membrane

Which best response would the nurse make to a client with antisocial personality disorder who asks, 'Can I have your phone number so I can call you for a date?' A. 'We are not permitted to date clients' B. 'It is against my professional ethics to date clients' C. 'I'm glad you like me, but I can't give out my phone number' D. Our relationship is professional; therefore, I will not see you socially'

D Rationale: The nurse would state, 'Our relationship is professional; therefore, I will not see you socially.' This response sets clear limits on the relationship and maintains a professional rather than a social role. Saying, 'We are not permitted to date clients,' shifts responsibility from the issue at hand to the institution. Stating, 'It is against my professional ethics to date clients,' avoids the real issue and shifts responsibility to the ethical code. Saying, 'I'm glad you like me, but I can't give out my phone number,' does not clarify the nature of the relationship as professional

An adolescent with leukemia is to be given a chemotherapeutic agent. Which time is best for the nurse to administer the prescribed antiemetic? A. As nausea occurs B. An hour before meals C. Just before each meal is eaten D. Before each dose of chemotherapy

D Rationale: The purpose of an antiemetic before chemotherapy is to prevent the child from experiencing nausea during and after the administration of the medication. Waiting until nausea has occurred is too late; the medication should be given before nausea occurs. The meals are not causing the nausea; the nausea is caused by the chemotherapy, and if nausea is not prevented, the child will not eat.

Which rationale will prompt the nurse to ask the provider for a different form of metformin when an older adult asks that extended-release metformin be crushed because it is difficult to swallow? A. This medication has a wax matrix frame that is difficult to crush B. The medication has an unpleasant taste, which most client find intolerable if crushed C. If crushed, this medication irritates mucosal tissue and can cause oral and esophageal ulcer formation. D. Extended-release formulations are designed to be released slowly and crushing the tablet will prevent this form occurring

D Rationale: The slow-release formulary will be compromised, and the client will not receive the entire dose if it is chewed or crushed. The capsules are not difficult to crush. Although taste could be a factor, this medication should not be crushed. Irritation of the mucosal tissue is not the reason the medication should not be crushed; however, this medication should be given with meals to prevent gastrointestinal irritation.

Which physiological factor increases the risk of falls for a toddler? A. Poor muscle coordination B. Undeveloped proprioception C. Underdeveloped skeletal muscles D. Poorly developed depth perception

D Rationale: The toddler has poor depth perception, which increases the risk of falling. Poor muscle coordination, undeveloped proprioception, or underdeveloped skeletal muscles are not physiological findings in a toddler.

The vital sign assessment of a client who gave birth reveal temperature 99.3; pulse 80 beats per minute, regular and strong; respirations 16 breaths per minute, slow and even; and blood pressure 148/92 mm Hg. Which vital sign would the nurse check more frequently? A. Pulse B. Respirations C. Temperature D. Blood pressure

D Rationale: This blood pressure is elevated. An elevated blood pressure can be indicative of preeclampsia, which can occur in the postpartum period. Careful evaluation would be needed. A pulse of 80 beats per minute is within expected limits. A respiratory rate of 16 breaths per minute is within expected limits. The temperature of 99.3 is slightly high but consistent with the physiology of the birthing process

Which mechanism would a nurse recall when caring for a client prescribed bisacodyl for constipation? A. Producing bulk B. Softening feces C. Lubricating feces D. Stimulating peristalsis

D Rationale: Bisacodyl stimulates nerve endings in the intestinal mucosa, precipitating a bowel movement. Bisacodyl is not a bulk cathartic. Bulk-forming laxatives, such as psyllium hydrophilic mucilloid, form soft, pliant bulk that promotes physiological peristalsis. Bisacodyl is not a stool softener. Stool softeners, such as docusate sodium, permit fat and water to penetrate feces, which softens and delays the drying of the feces. Bisacodyl is not an emollient. Emollient laxatives, such as mineral oil, lubricate the feces and decrease absorption of water from the intestinal tract

Which food or drink would the nurse instruct a client with a new colostomy to avoid because it produces large amounts of gas? A. Milk B. Cheese C. Coffee D. Cabbage

D Rationale: Cabbage is a gas-producing food that can cause a client with a colostomy problems with odor control and ballooning of the ostomy bad, which may break the device seal and allow leakage. Milk, cheese, and coffee should not cause excessive gas problems in moderation. The client with a new colostomy should slowly introduce new foods into the diet to test toleration

Which effect would the nurse anticipate after captopril is prescribed for a client? A. Increased urine output B. Decreased anxiety C. Improved sleep D. Decreased blood pressure

D Rationale: Captopril is an angiotensin-converting enzyme (ACE) inhibitor antihypertensive. It does not have diuretic, sedative, or hypnotic properties. Diuretics promote fluid excretion. Sedatives reduce muscle tension and anxiety. Hypnotics promote sleep

Which assessment is necessary for the nurse to complete in a client with chronic kidney disease receiving loop diuretics? A. Hemoglobin levels B. Occurrence of nausea C. Presence of constipation D. Intake and output measurement

D Rationale: Diuretics are administered to increase urine output, so the measure of intake and output are very important to diuretic use. Hemoglobin levels are important to monitor in the use of erythropoietin in the chronic kidney disease client. Nausea and constipation are important to monitor with the administration of iron-containing vitamins and mineral supplements

Which part of the renal system does furosemide exerts its effects? A. Distal tubule B. Collecting duct C. Glomerulus of the nephron D. Loop of Henle

D Rationale: Furosemide acts in the ascending limb of the Loop of Henle in the kidney. Thiazides act in the distal tubule in the kidney. Potassium-sparing diuretics act in the collecting duct in the kidney. Plasma expanders, not diuretics, act in the glomerulus of the nephron in the kidney

Which response will the nurse provide when a client taking a loop diuretic asks the nurse why changing positions slowly is necessary? A. "Your high blood pressure may cause headaches with position changes." B. "You will experience potassium fluctuations that will affect your balance." C. "You will need to rush to the bathroom while on the diuretic and we want to ensure you are safe." D. "Your blood pressure may drop with position changes, leading to dizziness and risk of falling."

D Rationale: Loop diuretics can lead to dehydration and orthostatic hypotension, which can increase dizziness and the risk of falls. The client should change positions slowly to allow for blood pressure to compensate. Hypertension can cause headaches, but that is not the reason to change positions slowly. Potassium loss with loop diuretics is not the cause of orthostatic hypotension. Diuretics do increase urination, but orthostatic hypotension is the reason for dizziness and falls.

Which action is responsible for the therapeutic effect of docusate sodium? A. Lubricates the feces B. Creates an osmotic effect C. Stimulates motor activity D. Softens the feces

D Rationale: The detergent action of docusate sodium promotes the drawing of fluid into the stool, which softens the feces. Lubricating the feces in the gastrointestinal (GI) tract is the action of lubricant laxatives such as mineral oil. Creating an osmotic effect in the GI tract is the action of saline laxatives, such as magnesium hydroxide, or other osmotics, such as lactulose. Stimulating motor activity of the GI tract is the action of peristaltic stimulants, such as cascara

The nurse is transferring a client from the bed to the chair. Which action would the nurse take first during the transfer? A. Place the client in a semi-Fowler position B. Stand behind the client during the transfer C. Turn the chair so it faces the bed D. Instruct the client to dangle the legs

D Rationale: The nurse would place the client in high-Fowler position, or 80 to 90 degrees, and then assist the client to the side of the bed. Next, the nurse helps the client sit on the edge of the bed and then instruct the client to dangle the legs. The nurse then faces the client and places the chair next to and facing the head of the bed. The semi-Fowler, or 30 to 40 degrees, position is not high enough to get the client in a sitting position

Which action would the nurse take when collecting a urine sample from the toddler who is not toilet trained? Select all that apply. A. Squeeze urine from the diaper B. Place a hat under the toilet seat C. Convince the child to void in the unfamiliar receptacle D. Attach single-use bags over the child's urethral meatus E. Use terms for urination that the child can understand

D, E Rationale: The nurse would use special collection devices for infants or toddlers who are not toilet trained. A single-use bag with self-adhering material over the child's urethral meatus can be used in toddlers to collect urine. The nurse needs to use terms for urination such as "pee pee" that the child is able to understand. Urine should not be collected by squeezing urine from the diaper because the results may be inaccurate. A young child is often reluctant to void in unfamiliar receptacles and should not be forced to do so. A potty chair or specimen hat placed under the toilet seat is usually more effective for young children than toddlers


संबंधित स्टडी सेट्स

DEP 2004 - Ch.6 Middle Childhood Quiz

View Set

Fundamentals of Corporate Finance Chapter 3 Introduction to Corporate Finance

View Set

AIS ch 8, AIS CH. 12 Quiz, Chapter 13 Questions - AIS, AIS CH 10, AIS- Chapter 11, Acc 250 - 3, AIS Chapter 7, AIS Ch 4, AIS Chapter 15, f

View Set

Chapter 6 Econ 201 Multiple Choice

View Set

Clinical Chemistry: Adrenal Function

View Set

6: SELLING AND MARKETING RESEARCH

View Set